Vous êtes sur la page 1sur 32

Integrated Physiology Lab Con Guide Questions Batch 2010 arranged by A.

Abad
Following taken from PPT from Section A, B, C and D (Thanks to James Co, Jill D., Cesar, and Pasky for getting the ppt I didn’t get in class). Previous notes taken from Chok’s reviewer.
Props to all of ya’ll who hammered some questions during the shifting week: Florida Gators for National Championship in basketball and football. Go Gators. Thanks to Ryan Go for the
additional notes. Thanks to Melody for gathering A6-2010 together, Irish for doing some last min work, and the Fil-am Core of 2010. Also thanks to the Section A members who helped
out from our future neuro surgeon Pasky, da cute group, respiratory expert Nicole, and lastly the hidden ninja. What’s this music..it’s not the Oscars. Kidding…Good luck & God Bless.

Relationship between breathing and systolic blood pressure (1-3 by Jill Desquitado 4 by Ryan Chua)

1. Describe blood pressure waveform using an arterial catheter. Label the following; systolic blood pressure (SBP), diastolic blood pressure (DBP),
pulse pressure and mean arterial blood pressure (MAP).

Systolic BP: Max aortic pressure following ejection from the left ventricle (Psystolic), Corresponds to max arterial vol
Diastolic BP: Lowest pressure in the aorta just before the ventricle ejects blood into the aorta (Pdiastolic).
Mean Arterial Pressure (MAP): pressure of blood pumped out of the left ventricle into the arteries; equal to CO x TPR
Pulse Pressure: maximal change in aortic pressure during systole; SBP minus DBP

Arterial Catheter
An arterial catheter is a very small plastic tube placed in one of your blood vessels (an artery)

Systolic Blood Pressure


Maximum arterial pressure reached during the peak ventricular contraction

Diastolic Blood Pressure


Minimum arterial pressure that occurs just before ventricular ejection begins

Mean Arterial Pressure


It is defined as the average arterial pressure during a single cardiac cycle.

Pulse Pressure
Change in blood pressure seen during the contraction of the heart
Difference between systolic pressure and diastolic pressure

2. Review physiologic principle involved in the auscultatory method of blood pressure measurement.

Auscultatory Method
Practitioner listens with a stethoscope applied to the skin of the antecubital
space
Pressure in the bag exceeds the systolic pressure, brachial artery is occluded
No sounds are heard
When inflation pressure falls just below systolic pressure, a small spurt of
blood escapes the occluding pressure of the cuff

Slighttapping sounds “Korotkoff sounds”


Auscutatory Measurement of BP
More sensitive
More precise method of measuring systolic pressure
Permits diastolic pressure to be estimated

Korotkoff Sounds
Sounds heard through the stethoscope as the pressure cuff deflates.
First sounds = systolic pressure
Sounds cease to be heard once the cuff has deflated past the diastolic
pressure.

Phases of Korotkoff Sounds


Phase 1 How Blood Flows?
With the pressure cuff inflated to beyond the systolic pressure, the Fluids must be
artery is completely occluded and no blood can flow through it. incompressible
Consequently, no sounds are heard above the systolic pressure. Tube must be straight,
rigid, cylindrical
Phase II Must have a constant
This phase is characterized by a swishing sound, caused by the radius
swirling currents in the blood as the flow through the artery Ohm’s Law
increases. Q= ΔP/R

Phase III Q- Blood flow


There is a resumption of crisp tapping sounds ΔP- Pressure Difference
Between P1 and P2
Phase IV
The blood flow is becoming less turbulent. Interrelationship among Pressure, Flow, and Resistance
Two factors affecting flow in the blood vessel
Phase V Pressure difference between the two ends of the vessel
This is the point at which sounds cease to be heard all together. Impediment to blood flow through the vessel (Vascular Resistance)
 The blood flow has returned to normal and is now laminar.
Some medical practitioners choose to record this point as the Turbulence
diastolic pressure. Rate of Blood flow is great
Obstruction in a vessel
3. Describe how arterial systolic, diastolic, mean, and pulse pressure are affected by changes in preload, cardiac contractility, heart rate (HR),
arterial compliance, and total peripheral resistance (TPR).
contractility   SV   CO  MAP
•  AFTERLOAD  SV  CO  MAP
•  HR   CO   MAP
•  • >170-180 HR  CO  MAP
•  TPR   MAP
o DIAMETER   RESISTANCE
o DIAMETER   PRESSURE

Vascular Compliance
ability of a blood vessel wall to expand and contract passively with
changes in pressure
ability of a vessel to distend

∆V
C=
∆P
4. Based on the previous guide questions, give the reason why the systolic blood pressure is lower during inspiration compared to expiration.

1. Diaphragm moves down Net effect:


2. Increased volume of thoracic air space ↓ preload
3. Decreased intra-thoracic pressure ↓ Stroke Volume
4. Increased intra-abdominal pressure ↓ Cariac Output
5. Increased venous return ↓ Mean Arterial Pressure
6. Increased blood to right atrium
7. Increased pooling of blood
8. to pulmonary circulation
During Inspiration 9. Right ventricle distends
10. Interventricular septum bulges
11. into the left ventricle
12. Decreased blood in the
13. left ventricle
14. *Decreased preload
15. *Decreased stroke volume
16. *FALL in SYSTOLIC BLOOD PRESSURE.
1. Decreased volume of thoracic air space Net Effect:
2. Increased intra-thoracic pressure ↑ preload
3. Decreased intra-abdominal pressure ↑ Stroke Volume
4. Decreased venous return ↑ Cariac Output
5. Decreased blood to right atrium ↑ Mean Arterial Pressure
During Expiration 6. Decreased pooling of blood in the
7. pulmonary system
8. Increased blood in left heart
9. *Increased preload
10. *Increased stroke volume
11. *HIGHER SYSTOLIC PRESSURE

Inspiration: ↑ thoracic air space  ↓ intrathoracic P  ↑ intraabdominal P  ↑ venous return  ↓ systemic pressure  ↑ pulmonary pressure  ↑
blood volume in pulmonary circulation
Inspiration with bag: ↑ intrathoracic pressure  ↓ venous return  ↑ LA volume  ↑ CO ↑ systemic pressure, ↓ pulmonary pressure, ↓ blood volume
in pulmonary circulation
Expiration: ↑ LA volume  ↑ CO ↑ systemic pressure, ↓ pulmonary pressure, ↓ blood volume in pulmonary circulation

Nitroglycerine (5-6 by Ryan Chua, 7-9 by Jeffery Co/Irish, 10-12 by Micki Avancena)

5. Discuss the correct temporal relationship, the pressure, volume, heart sound, and ECG changes in the cardiac cycle. Identify the intervals of
isovolumic contraction, rapid ejection, reduced ejection, isovolumic relaxation, rapid ventricle filling, reduced ventricular filling and atrial
contraction.

• P wave – atrial depolarization & contraction  slight ↑ pressure in atria (a wave)


• QRS complex – ventricular depolarization – initiates ventricular contraction  ventricular
contraction  ventricular pressure rises
• a wave – A contraction, rise in atrial pressure during late diastolic atrial contraction
• c wave –ventricles begin to contract, due to slight backflow of blood into atrium at onset of
ventricular contraction and bulging of the AV valves backward toward the atria because of ↑ P in
ventricles, closure of the AV valves during systole
• v wave – occurs toward end of V contraction; due to slow flow of blood into atria from veins
while AV valves are closed during ventricular contraction, arises from atrial filling during systole
• Heart Sounds – normally result from valve closure
o S1 –closure of AV (atrioventricular - tricuspid and mitral) valves during systole
o S2 –closure of the semilunar (pulmonary and aortic) valves during diastole
o S3 and S4 – not normally heard, but S3 may be in children & young adults
o S3 – rapid passive V filling, when Ventricles begin to expand during diastole
o S4 – rapid active V filling, when Atria contract & empty in latter Ventricular diastole
• ↑ aortic P  aortic valve opens after mitral valve closes, and closes before the mitral valve
opens = aortic valve open for a shorter time than mitral valve.
• ECG – record of the electrical discharges that occur within the heart through the various phases
of the chain of electrical communication within the heart.
o P wave – A muscle depolarization slight ↑ pressure in atria (a wave)
o P-R Interval – P wave start to QRS complex start, time required for impulse conduction to time
of V depolarization. From the beginning of the P wave to the beginning of the QRS complex. It
reflects the time required for the conduction of the impulse through the atria, AV node,
common bundle and bundle branches up to the time of ventricular depolarization.
o QRS Interval –Q wave start to S wave end, reflects ventricular depolarization. From the beginning of the Q wave to the end of the S wave. It
reflects the depolarization of muscle fibers in the ventricles.
o S-T Segment –QRS end to T start, pause between V muscle firing & repolarization. between the end of the QRS interval and the beginning of
the T wave. It is the pause between ventricular muscle firing and ventricular muscle repolarization.
o T wave – V muscle cell repolarization
o Q-T Interval –QRS start to T end, duration of ventricular electrical activity. It represents the duration of ventricular electrical activity.
o U wave – occurs between T to P waves, not always seen; may be due to the repolarization of the bundle branches and purkinje fibers

Systole – contraction of heart to squeeze out blood.


Diastole – relaxation and filling

Normal heart conduction


SA node

AV node

Bundle of HIS

Bundle Branches

Purkinje Fibers

6. Define the following: ejection fraction (EF), end diastolic volume (EDV), end systolic volume (ESV), and stroke volume (SV). Predict the change in
ejection fraction that would result from a change in a) preload, b) after load, and c) contractility.

• End Diastolic Volume (EDV) – Volume at the end of diastole (ventricular filling)
• End Systolic Volume (ESV) – Volume at the end of systole (ventricular contraction)
• Stroke Volume (SV)= EDV – ESV – volume of blood pumped out of one ventricle of the heart in a single beat
• Ejection Fraction = SV - percent of end-diastolic ventricular volume that is ejected with each stroke, SV/EDV
EDV

• ↑ preload / contractility = ↑ SV  ↑ EF [DIRECT]


• ↑ afterload = ↑ resistance  ↓ SV  ↓ EF [INVERSE]

Effect on PV Loop

↑ preload = ↑ EDV ↑ EDP = RIGHT


↑ afterload = ↑ ESV ↑ ESP = RIGHT
↑ contractility = LEFT

PRELOAD
Refers to the amount of end-diastolic stretch on myocardial muscle fibers

Determined by the volume of blood filling the ventricle at end-diastole

 the greater the filling volume, the greater is the stretch
 load of the heart prior to contraction (EDV)
AFTERLOAD
 Refers to the sum of all the loads against which the muscle fibers of both ventricles must shorten in order to eject blood into the arterial
circulations
 load that the heart pumps against with

CONTRACTILITY
 Describes the inotropic state of the myocardium that relates to the velocity and extent of myocardial fiber shortening regardless of preload
and afterload
 performance of the heart at a given preload and afterload

Summary:
↑ PRELOAD ↓ PRELOAD
↑ stroke volume ↓ stroke volume
↑ ejection fraction ↓ ejection fraction

↑ CONTRACTILITY ↓ CONTRACTILITY
↑ stroke volume ↓ stroke volume
↑ ejection fraction ↓ ejection fraction

↑ AFTERLOAD ↓ AFTERLOAD
↓ stroke volume ↑ stroke volume
↓ ejection fraction ↑ ejection fraction

7. Discuss the normal pressure volume loop. Discuss the change in pressure-volume loops that would result from changes in a) afterload, b) preload,
and c) contractility

Cardiac Output = Stroke Volume x Heart Rate


CO = SV x HR ~ 5 liters/min (at rest)
CO = (EDV - ESV) x HR

where End-diastolic Volume (EDV), End - systolic Volume (ESV)

Pressure-Volume Loop

• Determinants of Cardiac Performance


• Preload (initial stretch)
• Afterload
• Contractility
• Heart rate

Intrinsic Regulation of Stroke Volume (SV): Pressure-Volume Loop

• Huxley’s Sliding Filament theory


• Frank Starling Relationship (Starling’s Law)
• The greater the volume of blood in the ventricle, the greater the
contraction.

Extrinsic Regulation of Stroke Volume (SV):


• Sympathetic stimulation or circulating catecholamines
o increase contractility, decrease ESV
• Parasympathetic stimulation
o decrease contractility, increase ESV

Extrinsic Regulation of HR

• Sympathetic innervention (norepinephrine) increases the rate of


depolarization, increases HR.
• Parasympathetic innervention (acetylcholine) decreases rate of
depolarization, decreases HR.
• Adrenal gland (catecholamines), increases HR.
• Thyroid gland (thyroid hormone), increases HR.

Effects of:
a. afterload. Afterload is the pressure in the aorta that the left ventricle
needs to overcome in order to eject blood out of the heart. So:
increasing afterload will increase CD of the PVL
b. Preload. Preload is proportional to end diastoloic volume (until threshold). And will increase stroke volume. An increase in preload will also
increase ABC of PVL
c. Contractility. Will increase DEF

Diastolic filling starts at A, when the mitral valve opens, and it terminates at C, when the mitral
valve closes. The initial decrease in left ventricular pressure (A to B), despite the rapid inflow of
blood from the left atrium, is attributed to progressive ventricular relaxation and distensibility.
During the remainder of diastole (B to C), the increase in ventricular pressure reflects ventricular
filling and the changes in the passive elastic characteristics of the ventricle. Note that only a
small increase in pressure accompanies the substantial increase in ventricular volume during
diastole (B to C). The small pressure increase reflects the compliance of the left ventricle during
diastole. The small increase in pressure just to the left of C is caused by the contribution of atrial
contraction to ventricular filling. With isovolumic contraction (C to D), pressure rises steeply, but
ventricular volume does not change because the mitral and aortic valves are both closed. At D,
the aortic valve opens, and during the first phase of ejection (rapid ejection, D to E), the large
reduction in volume is associated with a steady increase in ventricular pressure. This volume
reduction is followed by reduced ejection (E to F) and a small decrease in ventricular pressure.
The aortic valve closes at F, and this event is followed by isovolumic relaxation (F to A), which is
characterized by a sharp drop in pressure. Ventricular volume does not change during the interval
from F to A because the mitral and aortic valves are both closed. The mitral valve opens at A to
complete one cardiac cycle.

(Fig 16-10)
Figure 16-10 Left atrial, aortic, and left ventricular pressure pulses correlated in time with aortic flow, ventricular volume, heart sounds, venous
pulse, and the electrocardiogram for a complete cardiac cycle in the dog.

PRELOAD
• lRefers to the amount of end-diastolic stretch on myocardial muscle fibers
• lDetermined by the volume of blood filling the ventricle at end-diastole
• lthe greater the filling volume, the greater is the stretch

Increased preload in Left Ventricular Pressure Volume Loop ↑EDV


↑SV
↑EF
Decreased Preload in Left Ventricular Pressure Volume loop
↓SV
↓EF
↓ EDV

AFTERLOAD
lRefers to the sum of all the loads against which the muscle fibers of both ventricles must shorten in order to eject blood into the arterial
circulations
Increased afterload in Left Ventricular Pressure Volume Loop
↓SV ↑ SV
↓EF ↑ EF
↑ESV ↓ ESV

Decreased afterload in Left Ventricular


Pressure Volume Loop
CONTRACTILITY
Describes the inotropic state of the myocardium that relates to the velocity and extent of myocardial fiber shortening regardless of preload and
afterload

Increase in Contractility on Left Ventricular Pressure Volume Decrease in Contractility on Left Ventricular Pressure Volume Loop
Loop
↑ SV ↓ SV
↑ EF ↓EF
↓ ESV ↑ ESV

8. Discussed the work done by the cardiac muscle in relation to the normal pressure volume loop of the heart.

Please look at the Pressure Volume Loop in Question 7


ABC is diastole
CD is isovolumic contraction
DE is rapid ejection
EF is reduced ejection
FA is isovolumic relaxation
Stroke Work
 is the amount of energy that the heart conveys into work each heartbeat
 W = P x change in Volume
Change in volume
= product of aortic pressure and stroke volume
= EDV – ESV
Total Energy in one cardiac cycle
E = (P) (V) + (1/2mv²) + (k.T.Δt)

Tension heat
= product of wall tension and length of time
= major determinant of the total energy requirement

9. Describe the phasic flow of blood to the ventricular myocardium through an entire cardiac cycle. Contrast this cyclic variation in myocardial flow
a) in the walls of the right and left ventricles and b) in the subendocardium and subepicardium of the left ventricle. Identify the area of the ventricle
most susceptible to ischemic damage and why the risk is increased at high heart rates.

Extravascular compression (EC) is the temporary halt in blood flow to the ventricular walls
brought about by the compression of blood vessels by the myocardium during contraction.

As a result, blood flow to the ventricular walls is highest at diastole (ventricular relaxation)

Blood supply to the epicardium is found superficial to the thick muscles of the heart and so,
is not affected by EC.

Therefore, blood flow to the epicardium is highest during systole while blood flow to the
endocardium is highest at diastole.

Most susceptible to ischemia is the subendocardium

Increase in HR will increase contractions of the ventricular wall. Time for diastole will
decrease while systole will relatively increase making it more dangerous for the deeper
layers of the myocardium.
Normal Coronary Blood Flow
•Resting- 225 mL/min( 4-5% of total CO)
•Influenced by:
–Aortic pressure
–Extravascular compression
•Systole- ventricular contraction
•Diastole-ventricular relaxation

Figure 23-2 Pressure-flow relationships in the coronary vascular bed.


At constant aortic pressure, cardiac output, and heart rate, coronary
artery perfusion pressure was abruptly increased or decreased from
the control level indicated by the point where the two lines cross. The
closed circles represent the flows that were obtained immediately
after the change in perfusion pressure; the open circles represent the
steady-state flows at the new pressures. There is a tendency for flow
to return toward the control level (autoregulation of blood flow), and
this is most prominent over the intermediate pressure range (about
60 to 180 mm Hg). (From Berne RM, Rubio R: Coronary circulation.
In Handbook of physiology, sect 2, The cardiovascular system: the
heart, vol I, Bethesda, Md, 1979, American Physiological Society.

Contrast this cyclic variation in myocardial flow a.) in the walls of the right and left ventricles
LEFT HEART RIGHT HEART
Early Systole-great Systole-develop lower pressure
force of extravascular compression (blood flow is reversed) -constitutes a much greater proportion of
Early Diastole-maximal coronary inflow coronary inflow
 stronger compression of LEFT ventricular muscle Diastole

and b.) in the subendocardium and subepicardium of the left ventricle


SUBENDOCARDIUM SUBEPICARDIUM
-compressed due to the high intraventricular pressures -least ventricular myocardial pressure
-extra vessels-compensate
greatest extravascular compression

Identify the area of the ventricle most susceptible to ischemic damage and why the risk is increased at high heart rates
ISCHEMIA- insufficiency of blood supply
Acute coronary occlusion
Myocardial Infarction
SUBENDOCARDIAL SURFACE
-greatest: left ventricular myocardial pressure
-blood vessels are intensely compressed (systolic contraction) extra difficulty obtaining adequate blood flow

At increased heart rate:


time for diastole
blood perfusion
time for systole
period of restricted inflow
=mechanical reduction in mean coronary blood flow
*diminished delivery of Oxygen
and metabolic substrates

10. Compare the excitation contraction coupling of a cardiac and vascular smooth muscle.

CARDIAC MUSCLE CONTRACTION


VASCULAR SMOOTH MUSCLE CONTRACTION
1. Extracellular calcium from the interstitial fluid enters myocyte
via Ca++ channel 1. Increase in free intracellular Ca++
2. It acts as a trigger to release Ca++ from SR (CICR) 2. Free calcium binds to calmodulin
3. Concentration of free Ca++ increases 3. 4Ca++-calmodulin activates MLCK, an enzyme that is capable of
phosphorylating MLC in the presence of ATP
4. This Ca++ then binds to troponin C 4. MLC phosphorylation leads to cross-bridge formation between
5. Ca++ - troponin complex interacts with tropomyosin to unblock myosin and actin
active sites between the actin and myosin 5. Contraction would then occur
6. The unblocking initiates cross-bridge cycling hence contraction of 6. When free intracellular Ca++ levels fall below 10-7 mol/L, it
the myofibrils dissociates from calmodulin.
. 7. Calmodulin dissociates from the MLCK.
7. Relaxation occurs when free intracellular Ca++ levels fall below 8. MLCK is inactivated.
10-7 mol/L (Ca++ -Na+ exchanger; 1 Ca++ out: 3 Na+ in) 9. Thus MLC is dephosphorylated hence relaxation occurs.
CARDIAC MUSCLE VASCULAR SMOOTH MUSCLE

rapid and of relatively short duration contractions slow, sustained, tonic contractions
contains actin and myosin WITH troponin contains actin and myosin WITHOUT troponin
Free calcium in the cytoplasm binds to protein troponin C Free calcium binds to protein calmodulin
Ca++ - troponin complex interacts with tropomyosin to Ca++-calmodulin complex activates MLCK and
unblock active sites between actin and myosin phosphorylates MLC in the presence of ATP

The unblocking initiates cross-bridge cycling and hence MLC phosphorylation leads to cross-bridge formation
CONTRACTION occurs between myosin heads and actin and hence,
CONTRACTION occurs

has T-tubules, troponin, fast Na channels lacks T-tubules, troponin, fast Na channels
close association bet. Action potential and contraction no close association bet. Action potential and contraction
electromechanical coupling-Ca
pharmacomechanical coupling (predominant)

11. Discuss the receptors found in a vascular smooth muscle. By what mechanism can arterial smooth muscle can be relaxed and promote
vasodilatation.

RECEPTORS FOUND IN THE VASCULAR SMOOTH MUSCLE

•ALPHA ADRENERGIC RECEPTORS (constrictors)


•BETA ADRENERGIC RECEPTORS (dilators)
•ARTERIAL BARORECEPTORS
–Widened areas at the origins of the internal carotid arteries (CAROTID SINUS) and at the aortic arch (AORTIC SINUS)
–Responds to VASCULAR STRETCH and DEFORMATION induced by CHANGES IN ARTERIAL BLOOD PRESSURE
–ENHANCED by ↑ in ARTERIAL BLOOD PRESSURE
–DIMINISHED by ↓ in ARTERIAL BLOOD PRESSURE

•PERIPHERAL CHEMORECEPTORS
–Small highly vascular bodies
–Located in the region of the aortic arch (AORTIC BODIES) and just MEDIAL TO THE CAROTID SINUSES (CAROTID BODIES)
–Sensitive to changes in PO2, PCO2, pH of blood
- ↓ PaO2 → aortic and carotid body stimulation → VASOCONSTRICTION
- Increases tone and capacitance of vessels
- Can also be found in CARDIAC MUSCLES
–Transmit PRECORDIAL PAIN associated with INADEQUATE BLOOD SUPPLY TO THE MYOCARDIUM
- SIMULTANEOUS occurrence of HYPOXIA and HYPERCAPNIA à enhanced chemoreceptor stimulation

RELAXATION & VASODILATION OF SMOOTH MUSCLE

A.Myogenic Mechanism
•Vascular smooth muscle contracts in response to an increase in pressure difference across the blood vessel wall (transmural pressure) and relaxes in
response to decrease in transmural pressure

B.NITROGLYCERIN:
1. Denitrated by Glutathione-S-transferase
2. Nitrite release converts nitroglycerin to NITRIC OXIDE (NO)
3. NO activates guanylyl cyclase
4. Formation of cyclic guanosine monophosphate (cGMP)
5. cGMP activates protein kinase A
6. phosphorylation of MLCK with subsequent dephosphorylation of myosin light chain and Ca ion release
7. Relaxation = VASODILATION

Types of Receptors found in a Vascular Smooth Muscle (added from


another reviewer)

α-adrenergic receptors
- would be bound with
Norepinephrine (NE) activating the Vasodilatory and Vasoconstictory Mechanisms in Vascular Smooth Muscle
Phospholipase C causing formation 1. Autonomic nervous System
of IP3 w/c in turn stimulates Vasodilation: through Epinephrine
release of Ca2+ from SR ----- Vasoconstriction: through Norepinephrine
vasoconstriction
ß2-adrenergic receptor 2. Vascular Receptors
-would be bound with Epinephrine Vasodilation: ß2-adrenergic
resulting to inc. cAMP leading to stimulation, adenosine, and
dec. Ca2+ ----- vasodilation prostacyclin
AII receptor Vasoconstriction:α-adrenergic
-would be bound to AngiotensinII stimulation
activating the Phospholipase C causing formation of IP3 w/c 3. Messenger systems
in turn stimulates release of Ca2+ from SR ----- vasoconstriction Vasodilation: receptors via G proteins to Adenylate cyclase
forming cAMP until myosin-light chain kinase is inhibited
ETa receptor Vasoconstriction: recptors… G-proteins…phospholipase C… IP3 and PKC…
- would be bound to Endothelin-I contractile proteins
activating the Phospholipase C 4.Endothelium
causing the formation of IP3 w/c vasodilation: EDRF (NO)--guanylate cyclase—formation of cAMP
in turn stimulates release of Ca2+ vasoconstriction: through Endothelin
from SR ----- vasoconstriction

12. Summarize the results of the intervention with regards the MAP and HR. Give the physiologic explanation of the result. Discuss the changes in
the ejection fraction, end diastolic volume, end systolic volume, stroke volume and pressure volume loop with this intervention.

Nitroglycerine = relaxation of vascular smooth muscle by forming NO  ↑ cGMP  myosin light chain dephosphorylation
• ↓ HR
• ↓ MAP due to ↓ preload  ↓

•Mean arterial pressure •End Diastolic Volume


–Cardiac output (CO) x Total peripheral resistance (TPR) –Ventricular end diastolic pressure
•Heart rate (HR) •End Systolic Volume
–Number of cardiac cycles per minute (beats per minute) –Ventricular end systolic volume
•Ejection Fraction •Stroke Volume
- SV / EDV EDV - ESV

Effect of Nitroglycerin
–Vasodilation of central arteries and veins
–Decrease Mean Arterial Pressure
–Decrease Preload
–Increase After load
–Increase Contractility
–Increase Heart Rate
•ejection fraction decreased
•end diastolic volume decreased
•end systolic volume increased
•stroke volume decreased

Parameter Effect Explanation


Cardiac rate ↑ Compensatory reflex mechanism; due to a decrease in TPR which decreases peripheral venous vessel tone,decreasing
preload and subsequently SV and CO
MAP ↓ Due to decrease in TPR

Epinephrine (13-15 by Jai Batac 16-18 by Eden Castro)

13. Contrast the sympathetic and parasympathetic branches of the autonomic nervous system based on: spinal cord division of origin, length of
preganglionic and postganglionic neurons, neurotransmitters and receptors at the ganglionic and target organ synapse.

The autonomic nervous system includes the sympathetic and parasympathetic nervous system, wherein the sympathetic NS is for fright,
flight and fight response and the PNS for rest and digest mechanisms. Also, enteric NS comprises the ANS, which are intramural plexuses of the GI
tract. It receives input from SNS and PNS, but it can act independently.
SNS and PNS consist of cell bodies found in the CNS that will send its myelinated preganglionic fiber to the autonomic ganglion. From this
ganglion, an unmyelinated postganglionic fiber will supply the specific targets.

Sympathetic Parasympathetic
Pregang Postgang Pregang Postgang
Intermediolateral cell Prevertebral and CN III, VII, IX, X and sacral terminal ganglia in or near
SC origin (location of column in the thoracic and Paravertebral ganglion spinal cord (S2-4) target organ
neuron cell bodies) upper lumbar segments of
the SC
Length Shorter Longer Longer Shorter
NT Ach NE Ach Ach
1° postsynaptic receptor Nicotinic Adrenergic (α, β) Nicotinic Muscarinic
Myelination Yes No Yes No
14. Contrast epinephrine from norepinephrine with regards their site of release and affinity to receptors. Discuss the actions of the sympathetic
and parasympathetic system in the different organs.
 Norepinephrine
 Bulk is produced by sympathetic postganglionic fibers
 Effects are mediated directly by the sympathetic nervous system
 Epinephrine
 Produced exclusively by the adrenal medulla
 Effects are brought about exclusively by the adrenal medulla
 Secretion always accompanies a generalized sympathetic nervous system discharge
Norepinephrine is primarily produced by the postganglionic sympathetic fiber and is mediated by the sympathetic NS. Epinephrine, on the
other hand, is exclusively produced by the adrenal medulla and its secretion always accompanies a generalized sympathetic response. Also,
norepinephrine produced by subsequent action of different enzymes on tyrosine can be converted to epinephrine through the action of
phenylethanolamine N-methyltransferase, which uses S-adenosylmethionine as the methyl donor.
The endocrine cells of the adrenal medulla receive input fom sympathetic preganglionic neurons, are excited by acetylcholine, and release
catecholamines. However, the main catecholamine released is epinephrine. In humans, 80% of the catecholamine released by the adrenal medulla is
epinephrine and 20% is norepinephrine.

α receptors • Calorigenesis
 Vasoconstriction, dilation of the iris, contraction of the ADRENEGIC LOCATION AFFFINITY OF TYPICAL EXAMPLES OF
intestinal and bladder sphincter, and contraction of the RECEPTOR CATECHOLAMINE RESPONSE RESPONSES
pilomotor muscles TYPE FOR NE AND E ELICITED ELICITED
α1 receptors Postsynaptic; NE > E Excitatory Generalized
 Located postsynaptically α1 Most arteriolar
 effects are mediated by activation of the inositol triphosphate/ sympathetic vasoconstriction
diacylglycerol second messenger system target cells (↑ smooth
muscle
 Ca along with PKC mediates the hormone effects
contraction)
α2 receptors Presynatic or NE > E Inhibitory Decreased
 either presynaptic or postsynaptic α2 postsynaptic; motility in
 autoreceptors – receptors located presynaptically; inhibit Digestive digestive tract
transmitter release system (↓smooth
 decrease rate of synthesis of cAMP though an action on a G muscle
protein contraction)
β receptors Heart NE = E Excitatory Increased rate
 Subdivided on the basis of the ability of antagonists to block β1 and strength of
them cardiac muscle
 Protein that make up the receptors is composed of seven contraction
membrane spanning regions connected by intracellular and Skeletal E only Inhibitory Breakdown of
extracellular domains β2 muscle; glycogen in
smooth skeletal
muscle of muscle;
 Antagonized by the action of α1 receptors
some blood bronchiolar
β1 receptors
vessels and dilation and
• Increase heart rate organs arteriolar
• Increase heart contraction vasodilation in
β2 receptors skeletal muscle
• Skeletal muscle vasodilation and heart (↓
• Glycogenolysis smooth muscle
contraction)
• Bronchodilation
• Uterine relaxation
ORGAN Sympathetic Parasympathetic
Action Receptor Action Receptor
Eye
Iris Contracts α
Radial muscle - Contracts M3
Circular muscle Relaxes β Contracts M3
Ciliary muscle
Heart
SA node Accelerates β1> β2 Decelerates M2
Ectopic Pacemakers Accelerates β1> β2
Contractility Increases β1> β2 Decelerates M2
Blood Vessels
Skin, splanchnic Contracts α1
Skeletal muscle Relaxes β2
(Contracts) α1
Relaxes M3
Endothelium Releases EDRF ((NO)((NO) M3
Bronchiolar smooth muscle Relaxes β2 Contracts M3
GI
Smooth muscle
Walls Relaxes α2, β2 Contracts M3
Sphincters Contracts α1 Relaxes M3
Secretion Increases M3
Myenteric plexus Activates M1
GU
Bladder wall Relaxes β2 Contracts M3
Sphincter Contracts α1 Relaxes M3
Uterus, pregnant Relaxes β2
Contracts α Contracts M3
Penis, seminal vesicles Ejaculation α Erection M1
Skin
Pilomotor smooth Muscle Contracts
Sweat glands
Thermoregulatory Increases
Apocrine (stress) Increases
Metabolic
Liver Gluconeogenesis β2, α
Liver Glycogenolysis β2, α
Fat cells Lipolysis β3
Kidney Renin release β1
Autonomic nerve endings
Sympathetic Decreases NE release M
Pasympathetic Decrease Ach release α
15. Briefly describe the different signal transduction pathways. Discuss in detail which of these pathways is utilized by epinephrine and
neostigmine.

G-protein
Tyrosine Kinase Nitric Oxide ANP
cAMP Phosphoinositol Arachidonic acid
1° Messenger NE Ach Histamine
Receptor β-adrenergic mAchR Histamine TK receptor
Transducer Gα Gα Gβγ Dimerization
1° Effector AC PLC PLA2 Ras-activator protein GC GC
2° Messenger cAMP IP3 / DAG Arachidonic acid Ras protein cGMP cGMP
cAMP- Lipooxygenases, cGMP-dependent
2° Effector Ca release / PKC MAP kinases CHON kinases
dependent PK cyclooxygenase protein kinase
Paracrine mediator; From atrial cells,
Used by EGF, FGF, released by the
Notes Gs stimulatory, Gi inhibitory in response to ↑
NGF, BDGF & insulin endothelial cell;
membrane permeable atrial pressure
• Epinephrine – uses β-adrenergic receptor  activates AC  ↑ cAMP
• Neostigmine – anti-Achase  prolongs Ach effect

16. State the relationship between blood flow, velocity and cross sectional area of a blood vessel. Discuss Ohm’s law and Reynold’s number.
V = Q/A where Q= blood flow; A= cross-sectional area
↑ velocity = ↑ blood flow ↓cross-sectional area

Ex. Blood flow velocity is higher in the aorta (small cross-sectional area) than in the sum of all the capillaries (large cross-sectional area). The lower
velocity of blood flow in the capillaries optimizes conditions for exchange of substances across the capillary wall.

REYNOLD’S NUMBER – measure of the tendency for turbulence to occur.


Re = VD/ η/p where V = velocity of blood flow; D= diameter η= viscosity p=density
• Blood flow is laminar when Re is below 2000 and turbulent when Re>3000
• Turbulence occurs when
1) radius is large(aorta)
2) ↑ velocity (cardiac output)
3) ↓ low viscosity (anemia,↓hematocrit)

OHM’s LAW – Q = ∆P where: Q = Blood flow; ∆P = pressure difference between the two ends of the vessel; R= resistance
R
↑ flow = ↑ pressure ↓resistance

Velocity (v) - Rate of displacement of a particular


fluid with respect to time
- measured in cm/sec

Flow (Q) - Rate of volume of fluid per unit time


- measured in cm3/sec
Resistance (R) - Impediment to blood flow through a vessel. It occurs as a result of friction along the inside of the vessel
Flow, Velocity, X-sectional area

Ohm’s Law flow


• Q= P and tube diameter, which is all divided by the viscosity of the fluid. If
R Reynold's number exceeds an arbitrary value of 2000, it is highly probable
Where: that a turbulent flow pattern is present. The presence of turbulent flow
Q = Blood flow requires a greater driving pressure to generate a given airflow than is
P = Pressure difference between P1 and P2 necessary with laminar flow. Or, frictional resistance to airflow is greater
with a turbulent than laminar flow pattern.
Reynold’s Number Laminar flow
Re = pVD • Re is below 2000
n • movement is parallel to axis of tube
where: • air moves as a series of individual layers
V = velocity of blood flow • each layer moves at a different velocity from neighboring
D = diameter layers
n = viscosity • parabolic velocity profile
p = density Turbulent flow
The transition from laminar to turbulent flow is predicted by Reynold's • Re exceeds 3000
number. Reynold's number is a dimensionless number • irregular motions of flow elements through the tube
obtained by multiplying the density of the fluid by the linear velocity of • not confined to a definite laminae
• presence of rapid, radial, and circumferential mixing – α2 Epi, Gαi, AC, cAMP
• Greater pressure required – β 1-3 Epi, Gαs, AC, cAMP
Acetylcholine
Epinephrine • Cholinergic receptors
• utilizes the G-protein linked pathway – N1, N2
• adrenergic receptors:
– M1, M3, M5 Gαq, PLC, IP3 and DAG
– α1 norE, Gαq , PLC, IP3 and DAG
– M2, M4 Gαi, AC, cAMP

17. Explain how Poiseuille’s Law influences resistance to flow. What will be the effect of epinephrine on these variables (blood flow, velocity and
cross sectional area of a blood vessel)? Will there be turbulent flow in the large blood vessels? Why?

• Blood Flow (cm2) – rate of volume of fluid per unit time


• Velocity (cm/sec) – rate of displacement of a particular fluid with respect to time
• Resistance – Impediment to blood flow in a vessel due to friction along the inside of vessel
• Cross-sectional Area: Veins > Arteries
o ↓ cross-sectional area = ↑ velocity
VESSEL CROSS-SECTIONAL AREA (cm2)
Aorta 2.5
Venae Cavae 8
Small arteries 20
Arterioles 40
Small veins 80
Venules 250
Capillaries 2500
• Poiseuille’s Law (for steady laminar flow)
o F = ∆P π R4
o 8ηL
o Where F=flow; ∆P= driving pressure; R= inner radius; η= viscosity; l= length
o ↑ flow = ↑ diving pressure, ↑ radius, ↓ length, ↓ viscosity
↑Resistance =↑ viscosity, length ↓ R4

R4 = this is powerful. For example if blood vessel radius decreases by a factor of 2 then resistance increases by a factor of 16, and blood flow
decreases by a factor of 16.

• Epinephrine Action on blood vessels  ↑ resistance


o α-receptors (skin and muscle)  contraction
o β2-receptors (muscle)  relaxation

• Turbulent Flow
o Reynold’s Number (RE): measure of the tendency for turbulence to occur
o Turbulence occurs when: radius is large (aorta), ↑ Blood flow, RE > 3000
18. Summarize the results of the intervention with regards the pupils’ size, HR, MAP and intestinal movement. Discuss the physiologic basis for the
results.

Epinephrine causes sympathetic responses: ↑ pupil size, ↑ HR, ↑ MAP, ↓ intestinal movement
Parameter Receptors 2° Messenger Effect
Pupil size β Dilate papillary dilator muscle
HR β1 cAMP ↑ Phase 4 depolarization  ↑ conduction velocity 
↓ PR interval  Accelerate
MAP α2 IP3/DAG Vasoconstriction  ↑ TPR  MAP
Intestinal Movement α2β2 cAMP Inhibited

Neostigmine (19-21 by Cesar Palana 22 by Marryann Chiombon, 23 by Donnabelle Chu 24 by Maylyn Cauilan)

19. Discuss the events involved in the synaptic transmission between a parasympathetic postganglionic axon and a target organ. Relate
neostigmine’s mechanism of action in your discussion.

SYMPATHETIC PARASYMPATHETIC
PRE POST POST PRE
ganglionic Ganglionic ganglionic ganglionic

Cholinergic Noradrenergic Cholinergic Cholinergic


Nicotinic Adrenergic Muscarinic Nicotinic

Sequence: AP spreads over terminal fibers  Depolarization  ↑ Ca permeability  Ca2+


ions diffuse into nerve terminals  Ca2+ ions interact with secretory vesicles that are
adjacent to membrane  Secretory vesicles fuse with membrane  Ca-mediated
exocytosis  Ach secretion  Ach binds to muscarinic receptors on effector cells/organs
 Hydrolysis of Ach by Ach esterase
Neostigmine: Anti-Achase  prolongs Ach action

20. Discuss a typical action potential in a ventricular muscle and a pacemaker cell, labeling both the voltage and time axes accurately. Describe
how ionic currents contribute to the four phases of the cardiac action potential.

Pacemaker cells (SA node)


-Slow response
Conducting cell (Ventricular muscle)
-Fast response
Contracting cells (Atrium & Ventricle)
-Fast response
IONIC BASIS FOR FAST RESPONSE TYPE of AP
Phase 1° Ion Channel Ion Flow Negativity of MP
0 Depolarization Na open Na influx ↓
1 Rapid repolarization Na close K efflux ↑
2 Plateau Ca open, K open Ca influx, K efflux 0
3 Gradual repolarization Ca close, K open K efflux ↑
Non-permeability of protein anion, Na-K pump
4 RMP
(3 Na out + 2 K in), K leak channels

IONIC BASIS FOR SLOW RESPONSE TYPE of AP


Phase Feature Ion Channels Involved Ion Flow Negativity of
Membrane Potential

0 Less steep Slow Ca channels Ca Influx ê


No overshoot
1 Absent - - -
2 Less sustained Ca channels close rapidly K efflux é
K channels- OPEN
3 Gradual Repolarization Ca channels- CLOSE K efflux é
K channels- OPEN
4 Less negative RMP Less leaky K channels Less K efflux Less negative
(-60 to -70 mV)

Pacemaker vs Ventricular AP
Phase Notes
upstroke is less steep, SA & AV node do not have fast Na channels, voltage-sensitive Ca2+ channels open when threshold is reached,
0
depolarization mainly caused by slow influx of Ca2, low overshoot
1 early repolarization is absent
2 less sustained plateau phase
More gradual repolarization, Ca2+ channels rapidly inactivated soon after opened, ↓ Na+ permeability, ↑ K+ permeability, efflux
3
slowly repolarizes the cell
4 resting membrane potential (-55 to -60 mV [less negative]) due to leaky Na+ and Ca2+ channels

• Epinephrine = ↑ cAMP  enhanced Ca2+ channels (Phase 2 of ventricular; Phase 0 of pacemaker)  more rapid Ca2+ entry  rapid AP &
contractions
• Neostigmine = ↑ Ach  Ca2+ channel less likely to open (Phase 2) slower depolarization  cell less excitable
o K+ channels remain open (Phase 3) hyperpolarization  cell less excitable
21. Discuss the electrical attributes of action potential which determine the cardiac cell’s automaticity and conduction velocity.

P4 slope; (-) TP= AUTOMATICITY Conduction Velocity


(-) RMP It is the ability of the cardiac cell to conduct action
• Increased Phase 4 slope potential
• More negative threshold potential The characteristics of conduction differ in fast- and
• Less negative RMP slow-response fibers
Factors that Determine Conduction Velocity
Conditions that Alter Automaticity • amplitude of the action potential
CD; Ca++ ; S; E; T= P4 slope= Automaticity • slope of phase 0
O; Ach • negativity of the RMP
Increase Automaticity:
• Sympathetic activity P0 slope; (-) RMP; Amplitude = CV
• Epinephrine 1
• Hyperthermia Sympathetic; Ca++= P0 slope= CV
• Hypercalcemia Parasympathetic
• Hypercapnia 1 = (-) RMP= CV
Decrease Automaticity: Ischemia; K+
• Parasympathetic activity Calcium = Amplitude= CV
• Hyperoxemia 1

22. Which phase or phases in the action potential can be altered by the neostigmine and epinephrine to produce an effect in the cardiac rate?

Automaticity • The ability of cardiac cell to generate a spontaneous action


potential Conduction Velocity (CV)
• SA and AV nodes • The ability of the cardiac cell to conduct an action potential
• Conducting cells
Automaticity = P4 = CD; Ca++; S; E; T Net effect:
epinephrine Ach; O2 Tachycardia
CV = P0 = Sympathetic; Ca++
Parasympathetic
Neostigmine =Ach Automaticity = P4 = CD; Ca++; S; E; T Net effect:
-Cholinesterase inhibitor Ach; O2 Bradycardia
- Prolonged effects of Ach CV = P0 = Sympathetic; Ca++
Parasympathetic

23. Discuss the physiologic events that correspond to the different waves and intervals that are seen in a 12 lead ECG. Which of these ECG waves
and intervals can be altered by neostigmine?

• Atrial depolarization – P wave


• Transmission of electrical impulse from SA node through conduction system – PR interval
• Septal, Apical, Free Wall and Posterobasal Depolarization – QRS
• Transition from depolarization to repolarization – QT Interval
• Ventricular repolarization – T wave

Neostigmine causes bradycardia and the following are seen in the ECG waves and intervals:

• Heart rate: less than 60 bpm (40 – 59 bpm)


• P wave: upright, consistent, and normal in morphology and duration
• PR interval: within normal limits (between 0.12- 0.20 seconds)
• QRS: Normal
• T- wave: normal
• Rhythm: regular

24. Summarize the results of the intervention with regards the pupils’ size, HR, MAP and intestinal movement. Discuss the physiologic basis for the
results.

Neostigmine = Parasympathetic
Parameter Effect Explanation
Pupil size ↓ Ach + M3  ↑ IP3/DAG  ↑ Ca  contraction
HR ↓ Ach + M2  ↓ AC  ↓ cAMP  ↓ contractility
MAP ↓ Ach + M  ↑ Ca  ↑ NO  ↑ cGMP  vasodilatation
Intestinal Movement ↑ Ach + M3  ↑ IP3/DAG  ↑ Ca  contraction

Sciatic Nerve Stimulation (25-30 by Aurencio Abad)

25. Stimulation of the sciatic nerve produced movements of the leg. Explain the physiologic events that occur from the motor fibers stimulation
till the contraction of the leg skeletal muscles.

Generation of Action Potential


• Depolarization
o During rising phase of AP, Na+ channels open and Na+ rushes into the cell.
o Membrane becomes more permeable to Na+ than to K+, so that membrane
potential approaches Na+ equilibrium potential (+54 mV in squid axon).
o In squid giant axon, the peak membrane potential of the AP reaches about +40
mV
• Sodium channel inactivation
o Each Na+ channel closes a short time (~ 1 ms) after it opens, even though the
membrane remains depolarized
o Na+ inactiviation allows the cell membrane to recover from influx of positivie
charge.
• Repolarization
o Depolarization also triggers opening of K+ channels, but they begin to open
slightly later than Na+ channel
o Thus the peak inward flow of Na+ ions precedes peak outward flow of K+ ions
• Afterhyperpolarization (AHP)
o K+ channels do not instantly respond to membrane potential, and some remain
open when resting potential is reached
o K+ will continue to diffuse out of cell leading to a more negative (hyperpolarized) membrane voltage

Excitation-Contraction Coupling
Sequence: AP spreads over motor axon terminals Ca Influx  Ach secretion  Nicotinic receptor stimulation  Na influx Depolarization 
Dihydropyridine stimulation Ryanodine opening Ca release  Muscle contraction

26. Differentiate the two types of motor unit. Which type of motor unit is usually recruited when the sciatic nerve is stimulated by low intensity of
electrical stimulus?

Motor Unit Low intensity = Type I


• Functional contractile unit of a muscle
• Composed of:
• Anterior horn cell
• Axon (motor nerve)
• Muscle fibers (innervated by the anterior horn
cells)
Motor unit classification Number of fibers few many
Characteristics Type I Type II Fiber Diameter moderate large
Properties of nerve Force of unit low high
Cell diameter small large Metabolic profile oxidative glycolytic
Conduction velocity fast very fast Contraction velocity moderate fast
Excitability high low Fatigability low high
Properties of muscle cells
27. Explain the general mechanism of sensory transduction (How sensory information from the receptors are transmitted and interpreted in the
cerebral cortex). Discuss the factors that can modify pain (Counterirritants, Gating of the dorsal horn and descending pathways. Trace the origin of
the sensory pathways conducted by the sciatic nerves.
Sensory Pathway
• A set of sensory neurons arranged in series
• Sequential elements:
o First order neuron
• Primary afferent neuron
• Peripheral endings form a sensory receptor
• Responds to a stimulus, transduces it, transmits encoded info to the CNS
• Soma often located in a dorsal root or a cranial nerve ganglion
o Second order neuron
• Located in the spinal cord or brainstem
• Receives information from first order neurons
• Transmits the info to the thalamus
• Via local neural processing circuits and biophysical properties of these neurons
• Axon typically crosses the midline to ascend to the thalamus
o Third order neuron
• Resides in one of the sensory nuclei of the thalamus
• Transforms info from second order neurons
• Via local circuits in the thalamus and intrinsic membrane properties of this neuron
• Transmits signals to the cerebral cortex
o Higher order neurons
• Located in appropriate sensory receiving areas of the cerebral cortex
• Processes info further
• Sensory info results in perception, which is a conscious awareness of the stimulus

Factors That Modify Pain


Sensory Homunculus
• Counterirritants
o Inhibition of central sensory pathways ↑ Number of receptors in ↑ size of
• Gating in the dorsal horn a particular part of the cortical
o Pain transmission may be prevented by inputs mediated by large myelinated fibers body receiving
o Pain transmission may be enhanced by inputs that are carried over by fine afferent area
fibers ↑ Convergence ↓ acuity
o Inhibitory interneurons in the superficial dorsal horn may serve as the gating
↑ Receptive field (size) ↓ acuity
mechanism
• Activation of descending pathways ↑ Density of sensory ↑ acuity
o Analgesia from morphine and enkephalin units
o Stimulation of periaqueductal gray of the midbrain
o Projections into the raphe magnus and related serotonergic nuclei
o Inhibits the dorsal horn
28. Summarize the results of the intervention with regards the pupils’ size, HR, MAP and intestinal movement. Discuss the physiologic basis for the
results.
Parameter Effect Explanation
Pupil size ↑ Dilates pupil activates adrenergic postganglionic sympathetic
HR ↑ β1 Adrenergic Receptor  ↑Automaticity Via Sympathetic  ↑ Cardiac Rate
MAP ↑ ↑Venous tone  ↑Blood Volume  ↑Preload  ↑SV & ↑HR  ↑CO & ↑ TPR  ↑MAP
β2 Adrenergic Receptors  Activates AC  ↑ cAMP  Activates myosin
Intestinal Movement ↓
phosphatase  Muscle relaxation  ↓ intestinal movement

• Intervention: Sciatic nerve (central end) stimulation


o Peripheral nerve originating from L4, L5, S1-S3
o Contains efferent and afferent fibers supplying the lower extremities
o Stimulation of the nerve elicits pain and stimulates the sympathetic nervous system.
29. Discuss the non chemical control of respiration.
• Respiratory control center
o Two main parts
• Ventilatory pattern generator
o Sets rhythmic pattern
• Integrator
o Process input from higher brain centers and chemoreceptors
o Controls rate and amplitude of ventilatory drive
o Originates from medulla
• DRG (Inspiratory)
• VRG (Inspiratory and Expiratory)
• Central chemoreceptors
• Peripheral chemoreceptors
• Pulmonary mechanoreceptors
Receptor Location Stimulus Response
Stretch Airway smooth muscle Inflation ↓ Inspiration time
↑ Expiration time
Hering-Breur Inflation Reflex
Stretch Airway smooth muscle Deflation ↓ Expiration time
Hering-Breur Deflation Reflex
Irritant Airway epithelial cells Foreign Substance Rapid shallow breathing, coughing,
bronchoconstriction, hypersecretion
• Peripheral receptors
o Stimulation causes increased inspiration.
o Pain receptors in muscles and skin.
o Proprioceptors in muscles tendons and joints.
o Muscle spindles of diaphragm and intercostal muscles.

30. Summarize the results of the intervention with regards the respiratory rate and intrapleural pressure.
Parameter Effect Explanation
Respiratory rate Pain and emotional stimuli  Afferents from limbic system and hypothalamus

 Ventilatory pattern Generator (Medulla)  Motor neurons of phrenic nerves and
Intrapleural pressure ↑ intercostal nerves  Contraction of the diaphragm and intercostal muscle

Saline (31 by Aurencio Abad 32 by Karen Amoloza)

31. Explain the sequence of events mediated by cardiopulmonary (volume) receptors & baroreceptors that occur after an acute increase or
decrease in arterial blood pressure. Include receptor response, afferent nerve activity, CNS integration, efferent nerve activity to the heart, kidney,
hypothalamus, and vasculature.

• Arterial Baroreceptors • Cardiopulmonary (Volume) Receptors


o Stretch-sensitive mechanoreceptors o Stretch-sensitive mechanoreceptors
o Stimulus: changes in blood pressure o Stimulus: changes in blood volume
o Location: Carotid Sinus and Aortic Arch o Location: atria, ventricles & pulmonary vessels
o Afferent nerve: CN IX & X o Afferent nerve: vagus nerve
o CNS integration: NTS o CNS integration: NTS

Receptor Heart Vasculature Hypothalamus Kidneys Net Effect Via NTS to stimulate/inhibit
↑ MAP ↑ Arterial ↓ HR Vasodilation ( ↓ Preload) ↓ AVP release ↑ Urine ↓ BP (+)Vagal Nuclei &
Baroreceptors ↓ SV Venodilation ( ↓ TPR) (Pituitary) Formation Nucleus Ambiguus  ↑
Parasympathetic
(-) Vasomotor Center ↓
Sympathetic
↓ MAP ↓ Arterial ↑ HR Vasoconstriction ( ↑ TPR) ↑ AVP release ↓ Urine ↑BP (-) Vagal Nuclei &
Baroreceptors ↑ SV Venoconstriction ( ↑ (Pituitary) Formation Nucleus Ambiguus  ↓
Preload) Parasympathetic
(+) Vasomotor Center ↑
Sympathetic
↑ MAP (↑ ↑Volume receptor ↑ HR Vasodilation ↓ AVP (Post. ↑ Na, H20 ↓ blood (-)Vasomotor Center  ↑
ECV) discharge ↑ ANP Pituitary) excretion volume (MAP) Sympathetic (SA Node) and ↓
Sympathetic
↓ MAP ↓Volume receptor ↑ HR Vasoconstriction ↑ AVP (Post. ↓ Na, H20 ↑ blood (+)Vasomotor Center  ↑
(↓ECV) discharge ↓ ANP Pituitary) excretion volume (MAP) Sympathetic

↑ MAP  stretch receptors (carotid & aorta)  brainstem  parasympathetic dominance  vagus  ↓ HR + vasodilatation
32. Summarize the results of the intervention with regards HR, MAP and urine output. Discuss the physiologic basis for the results.
Parameter Effect Explanation
HR Decrease Active baroreceptors, ↓ sympathetic, ↑ parasympathetic (SA Node)
MAP Increase ↑ ECV, ↑ preload, ↑ SV, ↑CO, ↑ MAP
Urine Output Increase ↓ RAAA & ADH + ↑ ANP & GFR

HR & MAP: Saline infusion  ↑ ECF volume but no ICF/ECF water shift = volume expansion  ↑ total blood volume  ↑ venous return  ↑ preload 
↑ stroke volume  ↑ cardiac output  ↑ MAP  reflex
Urine output: ↑ MAP  water-losing mode = ↓ RAAA & ADH + ↑ ANP & GFR  ↓ salt retention & ↑ water loss

• 0.9% Saline – isosmotic to plasma


o no change in plasma osmolality
o increase in ECF
o increase in vascular volume
o increase in ECV (Volume Expansion)
o increase in MAP  active baroreceptors
o active baroreceptors
• ↑ ANP (atrial myocytes)  ↑ Urine output
• ↓ AVP (posterior pituitary)
o Collecting Ducts  ↓ H2O permeability  ↑ Urine output  ↓ blood volume  ↓ SV  ↓ CO  ↓ MAP
• ↑ parasympathetic
o SA node  ↓ HR  ↓ CO  ↓ MAP
• ↓ sympathetic
o SA Node  ↓ HR
o Veins  venodilation  ↓ preload  ↓ SV  ↓ CO  ↓ MAP
o Arterioles  vasodilation  ↓ TPR
o Kidneys
• ↑ GFR + ↓ RAAA  ↑ Urine output  ↓ blood volume  ↓ SV  ↓ CO  ↓ MAP

Hemorrhage (33-34 by Karen Amoloza 35-36 by Bianca Arambulo 37-39 by Mike Castro 40 by Cathy Cifra)

33. Discuss and relate the mechanisms of hemostasis that come into play when vascular injury occurs.

Class I Hemorrhage involves up to <15% of blood volume


Class II Hemorrhage involves 15-30% of total blood volume Circulatory ↓ Blood Volume (ECF), ↓ DiastolicSystolic, ↓ Pulse pressure,
Class III Hemorrhage involves loss of 30-40% of circulating blood volume Collapsed veins, ↓ Cardiac Output (CO), ↓ RBC Concentration,
Class IV Hemorrhage involves loss of >40% of circulating blood volume Skin Paleness and Cyanotic, ↓ Nutrient transport
Respiratory ↓ Oxygen Concentration and transport
Nervous ↓ Nutrients and oxygen to the brain
Urinary ↓ Renal Blood Flow
Hemostasis – cessation of bleeding
5 mechanisms
 Vascular spasm
 Formation of platelet plug
 Formation of blood clot
 Clot retraction
 Lysis of the clot
Mechanism
• Vascular Spasm
o more traumatized the greater the spasm

Cause Effect Net Effect


Local Humoral Factors (platelets-Tromboxane A2) Generalized Arteriolar
Trauma Local myogenic spasm (direct damage of the vasucular wall) Vasoconstriction  ↑ MAP
Nervous reflex (pain nerve impulses  ↑ Heart rate & Myocardial
↓Blood volume/flow ↓ MAP  Recepor (Carotid Sinus & Aortic Arch)  ↓ Vagal Tone  ↑ Sympathetic tone Activity

• Formation of platelet plug o Mechanism


o Very small blood vessel • Formation of blood clot – blood coagulation
o Rather than blood clot
o Thrombocyte – megakaryocyte – no nuclei • Growth of fibrous tissue – to close the hole
o Active cytoplasmic factors: o Blood clot
• Ventilatory pattern generator o Larger vessels
• Actin and myosin molecules,thrombosthenin o 15 – 2- sec of trauma (severe), 1 -2 min. (minor)
(Contractile proteins)
• ER and Golgi – enzymes (thromboxane A2) & Ca+
• Mitochondria – ATP and ADP
• Enzyme system - Prostagladin
• Fibrin Stabilizing Factor
• Growth Hormone
o Glycoprotein – adhesion
o Phospholipids – Activating roles

34. Discuss the factors that control O2 transport and delivery to tissues? Speculate on what will happen to these factors during hemorrhage.

• Transport of oxygen in the blood


o 97%, combine loosely and reversibly to heme / 3% in plasma
• If PO2 is high – oxygen binds with hemoglobin (vise-versa)
• 95mmHg PO2 leaving the lungs / 40mmHg venous return
o Hemoglobin maintains the constant pressure in the tissues to facilitate transport (40mmHg – pressure gradient)
o Increase blood flow – increase oxygen transport
o Factors: (positive)
• ↑ Binding Capability
• ↑ Quantity
• ↓ Temperature
• ↑ pH
• ↓ pCO2
• ↓ 2,3 DPG
o Active cytoplasmic factors: Ventilatory pattern generator
o Determinants of O2 delivery to the tissues
• O2 Delivery = C.O. x {(PaO2 x 0.003ml/dl) + (Hg x 1.34 ml/dl x 02 saturation)}

Circulatory system Hematologic System Respiratory System

Hemorrhage
• Major Effects of Hypovolemia
• ↓ ECV
• ↓ MAP
• ↓ CO
• ↓ O2 delivery  Hypoxia
• Main body response: ↑ MAP, CO & O2 delivery
 Chemoreceptor Reflex
• ↓ RBC concentration  ↓ Hemoglobin  ↓ O2 transport  Hypoxia  O2 consumption > O2 perfusion
o Hyercapnia
o Hypoxia
o Activation of Chemoreceptors
• Response: ↑ Respiratory Rate

 Baroreceptor Reflex
• ↓ MAP  activation of baroreceptors (carotid & aortic baroreceptors)
• ↓ parasympathetic
• ↑ sympathetic activity
o Generalized arterial vasoconstriction
• ↑ HR and myocardial contractility
• ↑ MAP

 Sympathetic Nervous System (Brain ischemia)


 ↓ O2 delivery to the tissues + ↓ nutrients to the brain
• Vagal Centers  Sympathetic Nervous discharge
• Adrenal Glands  release of hormones for vasoconstriction
• ↑ O2 transport to the brain

35. What are the hormones are secreted during hemorrhage? Briefly state the rationale why these hormones are increased during times of stress.

1. Vasopressin (ADH): ↑ water reabsoprtion by the kidneys to = ↑ Plasma ADH=↑ Plasma Permeability Distal
increase blood pressure. It is actively secreted by Posterior tubules and Collecting ducts= ↑ H2O
Pituitary Gland in response to hemorrhage. The plasma Reabsorption
conc. of vasopressin rises progressively as arterial blood And less water is secreted
pressure diminishes. 2. Renin: secreted by the juxtaglomerular apparatus because of
Water deficit= ↑ Extracelular Osmolarity= ↑ ADH Secretion diminished renal blood flow during hemorrhagic hypotension.
Renin= ↑ Angiotensinogen  ↑ Angiotensin I  ↑
Angiotensin II by ACE
= ↑ bloop pressure
3. Aldosterone: from the adrenal cortex stimulates Na
reabsorption. The diminished blood flow raises levels of
Angiotensin II favoring the release of Aldosterone. Na is
actively reabsorbed, and water accompanies the Na
passively.

4. Erythropoietin: released from the kidney due to decreased


blood oxygen, stimulating the red bone marrow to increase
RBC production.
5. Catecholamines (Epinephrine and Norepinephrine) : ↓
sympathetic discharge  ↑ catecholamine release
• ↑ vasoconstriction

36. Differentiate the 3 different classes of hormones.

I. AMINE & PEPTIDE HORMONES


• Derivatives of amino acids Class of Major Location of Signalling Rate of
– Tyrosine: Thyroid Hormones; Catecholamines Hormone Form in Receptors Mechanisms Excretion
– Tryptophan: Melatonin; Serotonin Plasma
– Glutamic Acid: Histamine Second
• Derived from short chains of amino acids messengers
– TRH; vasopressin Amine & free plasma Enzyme Fast
• Derived from proteins Peptide membrane activation by
– Insulin; GH receptor
• Derived from glycoproteins Intrinsic
– LH; FSH; TSH enzyme
activity of
II.Steroids receptor
• Derived from cholesterol Steroids protein intracellular Intracellular Slow
– Cortisol bound receptors
– Testosterone directly alter
– Progesterone gene
– Estrogen transcription
– Aldosterone Eicosanoids free plasma Second Fast
membrane Messengers
III. Eicosanoids – Prostacyclins
• Derived from PUFA: Arachidonic acid – Leukotrienes
– Prostaglandins – Thromboxanes

37. Assuming that the dog and human physiology are the same and that the dog has body weight of 65Kg, estimate the a) total body water, b)
extracellular water, c) intracellular water, d) blood volume, and e) plasma volume. Identify normal extracellular fluid (plasma) osmolality and
concentrations of Na+, K+, Cl-, HCO3-, proteins, creatinine, and urea, and contrast these values with those for intracellular fluids. Identify major
routes and normal ranges for water intake and loss.

a) total body water = 0.6 X 65 kg body weight = 39 liter


B) extra cellular fluid = 0.2 X 65 kg body weight = 13L
c) Intracellular fluid = 0.4 X 65 kg = 26 L
d) blood volume = 3.25 L/(1 - 0.4) = 5.42 L
e) plasma volume = 0.25 X 13L ECF = 3.25 L

Plasma (mOsm/kg H2O) ICF (mOsm/kg H2O)


Na+ 142 14 Route ml/day
K+ 4.2 140 Water Intake
Cl- 108 4 Fluid 1200
HCO3- 24 10 In food 1000
Metabolically produced from food 300
Proteins 1.2 4
TOTAL 2500
Urea 4 4
Water Output
Creatinine 0.2 9
Insensible 700
Sweat 100
Feces 200
Urine 1500
TOTAL 2500
38. What are the factors that control glomerular filtration? Predict the changes in glomerular filtration caused by changes in filtration coefficient,
glomerular capillary hydrostatic pressure, glomerular capillary oncotic pressure, Bowman’s capsule hydrostatic pressure, efferent arteriolar
resistance and afferent arteriolar resistance.

DETERMINANTS OF ULTRAFILTRATION: GFR= Kf [(PGC-PBS)-σ (π GC- π BS)]

1. hydrostatic pressure Kf = filtration coefficient


2. oncotic pressure PGC = Glomerular capillary hydrostatic pressure
3. Kf (coefficient of filtration)- product of intrinsic πBS = Bowman’s space oncotic pressure
permeability of the glomerular capillary PBS = Bowman’s space hydrostatic pressure
and glomerular surface area πGC = Glomerular capillary oncotic pressure
σ = reflection coefficient
- resistance PGC and GFR
PGC is affected in three ways: • Changes in renal arteriolar pressure:
- blood pressure transiently PGC GFR
• Changes in afferent arteriolar resistance:
- resistance PGC and GFR mesangial cells (relaxed) ⇒ surface area, GF
• Changes in efferent arteriolar resistance: mesangial cells (contracted) ⇒ ↓ surface area, ↓ GF
plasma vasoactive agents (e.g. angiotensin II) ⇒ ↓ GF
↓ filtration coefficient and GF Net Filtration Pressure = PGC - PBS - πGC
membrane permeability determined by:
 molecular size and shape
 electrical charge
afferent arteriolar resistance ⇒ ↓ PGC and GF
↓ afferent arteriolar resistance ⇒ PGC and GF
↓ efferent arteriolar resistance ⇒ ↓ PGC and GF
efferent arteriolar resistance PGC and GF

renal arteriolar pressure ⇒ PGC and GF


↓ renal arteriolar pressure ⇒ ↓ PGC and GF

PBS ⇒ ↓ GF
↓ π GC ⇒ ↓ systemic plasma protein concentration,
GF
π GC ⇒ systemic plasma protein concentration,

39. Compare the distribution of cardiac output during hemorrhage and exercise.

Cardiac output is defined as the amount of blood that is pumped into increased cardiac contractility associated with the generalized
the aorta each minute by the heart. It also represents the quantity of increase in sympathetic neural activity.
blood that flows to the peripheral circulation; the cardiac output
transports substances to and from the tissues.
In this chart, it is determined by stroke volume and heart rate.

Stroke volume is the volume of blood that is ejected with each beat
while heart rate is the number of contractions of the cardiac
ventricles per unit time.

1) During exercise, there is an increase in cardiac output.


There is an increase in heart rate because this has a
permissive role in augmenting cardiac output during
physical exercise so as to increase perfusion to the skeletal
muscles. Stroke volume increases because there is an

During hemorrhage, cardiac output decreases because there is a decrease in stroke volume due to hypovolemia that happens during hemorrhage.

40. Summarize the results of the intervention with regards pupils size, HR, MAP and urine output. Discuss the physiologic basis for the results. What
are the compensatory mechanisms activated by hemorrhage?

Parameter Effect Explanation


2 muscles that control the size of the iris: Circular (parasymNS); Radial (SymNS)
Pupil size Increased  Symp stimulation of α1 ADRENERGIC RECEPTOR causes the contraction of the
radial muscle and subsequent dilation of the iris.
HR Increased  Sympathetic NS : sympathetic nerve fibers on heart; release of Epinephrine
MAP Decreased  ↓ Total blood volume; ↓ preload; ↓ Stroke volume; ↓ CO = ↓MAP
 Secretion of VASOPRESSIN (↓ Arterial BP = ↑ Vasopressin); Secretion of RENIN
Urine Output Decreased angiotensin II ; Release of aldosterone from adrenal cortex (Aldosterone =
reabsorption of water and sodium)

Hemorrhage = volume contraction  ↓ total blood volume  ↓ venous return  ↓ preload  ↓ SV  ↓ CO  ↓ MAP
(↑ HR to compensate)  ↑ RAA & ADH, ↓ ANP  water retention to maintain MAP = predominantly sympathetic response

Compensatory Mechanisms:
Baroreceptor Reflex: ↑ Sympathetic Tone; Arteriolar Constriction; Intense Renal Vasoconstriction:
Chemoreceptor Reflex: ↑ Existent peripheral vasoconstriction evoked by baroreceptor reflex
Cerebral Ischemia: Activation of sympathoadrenal system
Reabsorption of Tissue Fluids: ↓ Hydrostatic pressure in the capillaries; Reabsorption of interstitial fluid into the vascular compartment
Endogenous Vasoconstrictors: Cathecolamines; Vasopressin; Renin

Hypertonic NaCl (41-42 by by Cathy Cifra 43-44 by Melody Chua/Irish)

41. Describe the different feedback mechanisms involved in the maintenance of ECF volume and plasma osmolality?

• Plasma Osmolality - is a measure of the concentration of substances such as: Na+, K+, Cl-,urea, glucose, and other ions in the human blood
(normal: 285 – 295 mOsm/kg H2O)

Plasma osmolality = 2 plasma [Na] + [Glucose] + [Urea]


18 2.8
Unit: mOsm/kg Water

Two forces that determine fluid movement:


1. Hydrostatic pressure
2. Osmotic pressure

Control ECF Osmolality


Antidiuretic Hormone (Kidney: retain water, decrease urine
ouput; Sweat Glands: decrease water loss by
perspiration; Arterioles: contrict to increase BP)
 Hyperosmotic urine: water conserving mode
 ↑ permeability of collecting tubule to H2O and
urea; ↑ Water absorption in CD
 Hypoosmotic urine – water losing mode
 ↓ ADH  diuresis  hypoosmotic urine
 ↑ ADH  antidiuresis  hyperosmotic urine
 ↑ secretion; ↓ H2O excretion
 Regulators of ADH secretion:
 Osmolality of body fluids
 Volume and pressure of the vascular
system
 Nausea (stimulates)
 Antinatriuretic peptide (inhibits)
 Angiotensin II (stimulates)
o Reinin-angiotensin system (water and salt retention; ↑ Effective circulating volume; ↑ perfusion of Juxtaglomerular apparatus)
 ↑ secretion; ↓ NaCl excretion

o Thirst  Antero-lateral region of the hypothalamus  sensation of thirst


o Salt appetite
o Alteration in sweat content

Control of ECF Fluid Volume and Regulation of Renal NaCl Excretion


• Effective Circulating Volume (ECV)
– portion of the ECF volume that contained within the vascular system that is effectively perfusing the tissues
• directly proportional to:
– ECF volume (arterial and venous)
– Arterial blood pressure
– Cardiac output
Volume – Sensor Signals - ↑ GFR; arteriole dilation
• Vascular - ↓ Renin Secretion
– Low Pressure - ↓ Aldosterone secretion
• Cardiac atria - ↓ ADH secretion and action in CD
• Pulmonary Vasculature - ↑ NaCl and water excretion
– High Pressure
• Carotid Sinus Euvolemia
• Aortic Arch - Maintenance of Na+ Balance
• JG apparatus of kidneys - Volume Expansion
• Hepatic Positive Balance
• Central Nervous System Na Intake > Na excretion
o ↓ Sympathetic activity
Renal Sympathetic Nerves o ↑ ANP secretion
- ↓GFR; arteriole constriction
o Inhibition renin, ADH and aldosterone
- ↑ Renin Secretion; stimulate Macula densa
- ↑ PT, TAL, DT and CD NaCl reaborption o ↑ NaCl and Water excretion
- ↑ activity: ↓ NaCl excretion
- Volume Contraction
Negative Balance
Na Intake < Na excretion
o ↑ Sympathetic activity
o ↑ Renin, ADH and aldosterone
o Inhibit ANP
o ↓ NaCl and Water excretion
Atrial Natriuretic Peptide

42. Discuss the consequences in the different body water compartments if hypertonic and hypotonic solutions are infused intravenously.

Sixty percent of body weight consists of water, two-thirds of which is intracellular. Alterations in the concentration of salt, whether due to
primary changes in the amount of water or the amount of salt in the body can be of pathophysiologic importance. Note that sodium chloride is the
predominant solute in extracellular fluid (ECF). In contrast, cells contain mostly potassium, phosphate and proteins. This distribution of solutes
between cells and ECF is fundamental to the maintenance of normal cell function. Each solute has its own specific set of physiologic functions, yet
the total concentration of solute is also an important parameter.
The total amount of solute in a given amount of water is referred to as the osmolality of the solution. If two solutions of different osmolalities
are placed on different sides of a membrane that is permeable to solutes and water, water will move from the low to the high osmolality solution
while solutes will move in the opposite direction until both solutions have the same osmolality. Note that sodium, potassium, chloride and proteins
are not evenly distributed across the plasma membrane. This situation is more analogous to the experiment where two solutions of different
osmolalities are placed on different sides of a membrane that is permeable to water, but not to solutes. In this case, only water moves across the
membrane - which it continues to do until the osmolality is the same on both sides. The total amount of solute in a given amount of water that can
not cross a membrane freely is referred to as the tonicity of the solution. Now consider the cell: movement of most solutes across the plasma
membrane is restricted and water can move freely. Hence, it is the movement of water between intracellular fluid (ICF) and ECF that establishes
osmotic equilibrium across the plasma membrane.

The amount of intracellular solute is relatively constant under most conditions, however the amount of extracellular solute varies. This is so because
the major solutes that we ingest are sodium and chloride which are largely confined to the extracellular space. Hence, ECF and ICF tonicity and
volume are primarily dependent on the amounts of sodium and water in the body. This conclusion is illustrated by the following examples.

Suppose we give intravenous hypertonic sodium chloride (tonicity greater than that of the ECF):
(Osmolality = 1000 mOsm/kg)

ECF sodium concentration rises with hypertonic NaCl administration. Since sodium can not enter
cells, this draws water out of the intracellular space. Hence, hypertonic NaCl increases ECF volume
and sodium concentration/osmolality, while decreasing intracellular volume. And increase in water
movement in to the ECF.

At osmotic equilibrium
- Higher Osmolality for ECF and ICF than before infusion
- ↑Volume ECF
- ↓Volume ICF

Now suppose we give hypotonic NaCl (e.g. 1/2 normal saline or tonicity one-half that of normal ECF):

ECF sodium concentration falls  lowers ECF tonicity  causing water to move into cells  water
will stop moving into cells when ECF tonicity = ICF tonicity.
Hence, hypotonic fluid administration:
(Osmolality = 145 mOsm/kg)
a) lowers ECF sodium concentration/osmolality
b) increases cell water content/ICF
c) increases ECF volume.
Note that the increase in ECF volume is less than that seen with normal saline because some of the administered water moved into the cells.
At Osmotic Equilibrium:
- Lower osmolality for ECF and ICF before infusion
- Greater Volume in increase in ECF than ICF

43. Differentiate tubuloglomerular feedback and glomerulotubular balance.

Tuberoglomerular feedback mechanism

• Involved in the autoregulation of the GFR and RBF together with the Myogenic Mechanism
• Two parts of the mechanism:
o Afferent arteriolar mechanism
o Efferent arteriolar mechanism
(dependent of the juxtaglomerular complex)
• Juxtaglomerular complex consists of:
o Macula Densa Cells
• At the initial portion of the distal tubule
• Fx: senses the decrease of the NaCl concentration due to decrease in blood pressure. Effect:
• ↓ resistance of the afferent arterioles = ↑ GHP and GFR
(for it to become normal)

• ↑ Renin release = ↑ Angiotensin II formation


o ↑Angiotensin II = constrict efferent arterioles and ↑ GHP and GFR
( Note: The effector mechanism that affected the afferent arteriole resistance is most likely adenosine. It constricts the afferent arteriole (in
contrast to its vasodilator effect on most other vasculature beds). Adenosine triphosphate (ATP) and metabolites of arachidonic acid may contribute
to tubuloglomerular feedback (TGF). Nitric oxide (NO), a vasodilator produced by the macula densa, endothelial cells, and angiotensin II may also
play a role in TGF, but they are not essential for autoregulation.)
o Juxtaglomerular Cells

Glomerulotubular Balance
• The ability of the tubules to increase reabsorption rate in response to increased tubular load (increased tubular inflow)
• Ensures that the reabsorption rate of the proximal tubule is matched to the glomerular filtration rate.
• Helps to prevent overloading of distal tubular segments when GFR increases
• Refers to the fact that the total rate of reabsorption increases as the filtered load increases, even though the percentage of GFR
reabsorbed in the proximal tubule remains relatively constant at about 65 per cent
• it is independent of any hormones
• Two mechanisms:
(1)oncotic and hydrostatic pressures between the peritubular capillaries and the lateral intercellular space (i.e. Starling’s forces)
This protein-rich plasma leaves the glomerular capillaries, flows through the efferent arteriole, and enters the peritubular capillaries. The
increased oncotic pressure in the peritubular capillaries augments the movement of solute and fluid from the lateral intercellular space into the
peritubular capillaries. This action increases net solute and water reabsorption by the proximal tubule.

(2)increase in the filtered load of glucose and amino acids


o
As GFR and the filtered load of glucose and amino acids increase, Na+ and water reabsorption also rise.

Note: Other factors that may ↑ RBF and GFR:


 A high protein intake is known to increase both renal blood flow and GFR

↑ protein = ↑ release of amino acids into the blood (reabsorbed in the proximal tubule)
Amino Acids and sodium are reabsorbed together. Therefore there is a decrease delivery of NaCl to the Macula Densa. This causes the increase
GFR and RBF due to decrease resistance of afferent arteriole. But there is still increase excretion of waste products.

 Large increases in blood glucose levels in uncontrolled diabetes mellitus

Glucose is also reabsorb along with sodium. There is also decrease in the delivery of NaCl to the Macula Densa. Therefore there is
an increase in the GRF and RBF

TUBULOGLOMERULAR FEEDBACK GLOMERULOTUBULAR BALANCE


First line of defense Second line of defense

NaCl concentration dependent Tubular load


Early distal tubule Proximal tubule
Returns GFR and filtration of Na+ to normal values Matches reabsorption rate to GFR
Afferent and efferent arteriolar feedback mechanism Starling’s forces and increase in the filtered load of
glucose and amino acids

44. Summarize the results of the intervention. Give the physiologic basis for the results.

Parameter Effect
Cardiac rate Increase
MAP Increase
Urine output Decrease

• it is a hypertonic solution (Hypertonic NaCl)


• decrease urine output

Increase solute concentration is being reabsorbed by the proximal collecting tubule. Therefore by diffusion increases water reabsorption would
follow resulting to a decrease in the urine output.

Increase ECF volume

Antidiuretic Hormone
• Most important hormone that regulates water balance
• Synthesized in the supraoptic and paraventricular nuclei of the hypothalamus
• Secreted by the posterior pituitary

SOLUTE AND WATER REABSORPTION ALONG THE NEPHRON


In a quantitative sense, the reabsorption of NaCl and water represents the major function of the nephrons (approximately 25,000 mEq/day of Na+
and 179 L/day of water are reabsorbed). In addition, the transport of many other important solutes is linked either directly or indirectly to Na+
reabsorption.
Proximal Tubule The proximal tubule reabsorbs approximately 67% of the filtered water, Na+, Cl-, K+, and other solutes. In addition, the proximal
tubule reabsorbs virtually all the glucose and amino acids filtered by the glomerulus.
The key element in proximal tubule reabsorption is the Na+,K+-ATPase in the basolateral membrane. The reabsorption of every substance, including
water, is linked in some way to the operation of Na+,K+-ATPase. Na+ reabsorption. Na+ is reabsorbed by different mechanisms in the early (first half)
and late (second half) segments of the proximal tubule. In the early segment, Na+ is reabsorbed primarily with HCO3- and a number of organic
molecules (e.g., glucose , amino acids , Pi, lactate).
By contrast, in the second half of the proximal tubule, Na+ is reabsorbed mainly with Cl-. This difference exists because of differences in Na+
transport systems present in the early and late segments of the proximal tubule, and because of differences in the composition of tubular fluid at
these sites.
In the early segment of the proximal tubule, Na+ uptake into the cell is coupled with either H+ or organic solutes. Na+ entry into the cell across the
apical membrane is mediated by specific symporter and antiporter proteins, and not by diffusion through channels. For example, Na+ entry is
coupled with the pumping of H+ out of the cell by the Na+-H+ antiporter. H+ secretion results in NaHCO3 reabsorption. Na+ also enters proximal cells
by several symporter mechanisms, including Na+-glucose, Na+-amino acid, Na+-Pi, and Na+-lactate symporters. The glucose (and other organic
solutes) that enters the cell with Na+ leaves the cell across the basolateral membrane by passive transporter mechanisms. Any Na+ that enters the
cell across the apical membrane leaves the cell and enters the blood via the Na+,K+-ATPase.
In summary, in the early segment of the proximal tubule, the reabsorption of Na+ is coupled to that of HCO3- and a number of organic molecules.
Reabsorption of many organic molecules is so avid in this segment that they are almost completely removed from the tubular fluid. The reabsorption
of NaHCO3 and Na+-organic solutes across the proximal tubule establishes a transtubular osmotic gradient that provides the driving force for the
passive reabsorption of water by osmosis. Because more water than Cl- is reabsorbed in the early segment of the proximal tubule, the Cl-
concentration in tubular fluid rises along the length of the early proximal tubule.
In the second half of the proximal tubule, Na+ is primarily reabsorbed with Cl- across both the transcellular and paracellular pathways. Na+ is
reabsorbed with Cl- rather than with organic solutes or HCO3- as the accompanying anion. This occurs because the cells lining the late proximal
tubule have different Na+ transport mechanisms from those in the early proximal tubule. Furthermore, the tubular fluid that enters the late
proximal tubule contains very little glucose and amino acids but has a high concentration of Cl- (140 mEq/L) compared with that in the early
proximal tubule (105 mEq/L). The high Cl- concentration is due to the preferential reabsorption of Na+ with HCO3- and organic solutes in the early
proximal tubule.
Na+ enters the cell across the luminal membrane by the parallel operation of Na+-H+ and one or more Cl- anion antiporters. Because the secreted H+
and anion combine in the tubular fluid and reenter the cell, the operation of the Na+-H+ and Cl- anion antiporters is equivalent to NaCl uptake from
tubular fluid into the cell. Na+ leaves the cell by the action of Na+,K+-ATPase, and Cl- leaves the cell by the action of a KCl symport protein in the
basolateral membrane. NaCl is also reabsorbed across the late proximal tubule by a paracellular route.

Antidiuretic Hormone
• Two classes of stimuli that cause ADH release:
o osmotic stimuli (osmoreceptors)
o pressure stimuli (baroreceptors)
Physiology of ADH release

ADH is synthesized in the supraoptic and paraventricular nuclei of the hypothalamus. ADH is stored in granules which travel down the pituitary stalk
to the posterior pituitary where they are released under the appropriate stimuli. There are two classes of stimuli that cause ADH release: osmotic and
pressure stimuli. An increase in osmolality activates osmoreceptors that increase ADH release, while a decrease in pressure in the arterial or
venous side of the circulation activates baroreceptors which stimulate ADH release

During volume expansion, the volume sensors send signals to the kidneys, resulting in an increase in the excretion of NaCl and water. The signals
acting on the kidneys include
Decreased activity of the renal sympathetic nerves.
Release of ANP from atrial myocytes as well as other natriuretic peptides (e.g., urodilatin in the kidney).
Inhibition of ADH secretion from the posterior pituitary.
Decreased renin secretion, and thus decreased production of angiotensin II.
Decreased secretion of aldosterone, due to reduced angiotensin II levels, and elevated ANP levels.
The integrated response of the nephron to these signals is illustrated in Fig. 36-7. The important difference between a situation during volume expansion
and that during the euvolemic state is that the renal response is not limited to the collecting duct; rather, it involves the entire nephron.Three general
responses to volume expansion occur (Fig. 36-7). The numbers correlate to those encircled in the figure.
GFR increases. GFR increases primarily as a result of the decrease in sympathetic nerve activity. Sympathetic fibers innervate the afferent and efferent
arterioles of the glomerulus and control their diameter. Decreased sympathetic nerve activity leads to their dilation. Because this effect appears to be
greater on the afferent arteriole, the hydrostatic pressure within the glomerular capillary is increased, and GFR increases. ANP and urodilatin also
increase GFR by dilating the afferent and constricting the efferent arterioles. With the increase in GFR, the filtered load of Na+ increases.
Reabsorption of Na+ decreases in the proximal tubule. Several mechanisms appear to be involved in reducing Na+ reabsorption by the proximal tubule,
but the precise role of each of these mechanisms remains controversial. Because activation of the sympathetic nerve fibers innervating this nephron
segment stimulates Na+ reabsorption, the decreased sympathetic nerve activity resulting from volume expansion may contribute to the decreased Na+
reabsorption that occurs. In addition, angiotensin II directly stimulates Na+ reabsorption by the proximal tubule. Because angiotensin II levels are also
reduced under this condition, it is possible that proximal tubule Na+ reabsorption is decreased as a result. The increased hydrostatic pressure within the
glomerular capillaries also leads to an increase in the hydrostatic pressure within the peritubular capillaries. This alteration in the capillary Starling forces
reduces the absorption of solute (e.g., NaCl) and water from the lateral intercellular space, thus reducing tubular reabsorption (see Chapter 35 for the
mechanism).
Na+ reabsorption decreases in the collecting duct. Both the increase in the filtered load and the decrease in proximal tubule NaCl reabsorption result in
the delivery of large amounts of NaCl to Henle's loop and the distal tubule. Because increased activity of both sympathetic nerves and aldosterone
stimulates NaCl reabsorption by Henle's loop, the reduced nerve activity and low aldosterone levels seen with volume expansion could in theory reduce
NaCl reabsorption by this nephron segment. However, because reabsorption by the thick ascending limb is load dependent, these effects are offset, and
the fraction of the filtered load of Na+ reabsorbed by Henle's loop is actually increased. Nevertheless, the amount of Na+ delivered to the beginning of
the collecting duct is increased compared with the euvolemic state (Fig. 36-8).
The amount of Na+ delivered to the beginning of the collecting duct varies in proportion to the degree of volume expansion. This increased load of Na+
overwhelms the reabsorptive capacity of the collecting duct, which is even further reduced by the actions of ANP and urodilatin and by the decrease in
the circulating levels of aldosterone. The final component in the response to volume expansion is the excretion of water. As Na+ excretion increases,
plasma osmolality begins to fall. This results in decreased secretion of ADH. ADH secretion is also decreased in response to the elevated levels of ANP.
In addition, ANP inhibits the action of ADH on the collecting duct. Together, these effects decrease water reabsorption by the collecting duct, thereby
increasing water excretion by the kidneys. Thus, the excretion of Na+ and water occurs in concert; euvolemia is restored, and body fluid osmolality
remains constant. As already noted, the time course of this response (hours to days) depends on the magnitude of the volume expansion. Thus, if the
degree of volume expansion is small, the mechanisms just described will restore euvolemia within 24 hours. However, with large degrees of volume
expansion, the response can take several days.
In summary, the nephron's response to volume expansion involves the integrated action of all its parts. The filtered load is increased, proximal tubule
reabsorption is reduced (GFR is increased, whereas proximal reabsorption is decreased; thus, G-T balance does not occur under this condition), and the
delivery of NaCl to the beginning of the collecting duct is increased. This increased delivery, along with inhibition of collecting duct reabsorption, results
in the excretion of a larger fraction of the filtered load of Na+, thus restoring euvolemia.
2% Glucose (45 Melody Chua 46-48 by Danilyne Bambico)

45. Discuss the control of blood sugar.


• Absorption of Glucose
o Duodenum and upper jejunum has the highest capacity to absorb monosaccharide (glucose galactose and fructose)
• In the brain
o Utilize about 60-70% of body glucose
o Starvation: glucose and ketone bodies are used
o ↓ glucose  Hypothalamus  Adrenal Glands (EPINEPHRINE)  glucose from liver (GLYCOGENOLYSIS and
GLUCONEOGENESIS) 
increase glucose Hypothalamus  Gastrointestinal hormone (GLP-1 (GLUCAGON-LIKE PEPTIDE 1))  augments B cell
response to glucose  increase insulin synthesis  decrease food intake
• In the liver
o source of 90% of the body glucose
o Glycogenesis:
o ↑ glucose level
o intestinal absorption of glucose
o storage as GLYCOGEn
o Glycogenolysis & Gluconeogenesis
• ↓glucose level
• Glucogen --> G6PO4 --> glucose
(enzymes: phosphorylase; G-6-phosphatase)

COUNTER REGULATORY HORMONES


Glucagon:↑ glycogenolysis & gluconeogenesis
Epinephrine:↑ glycogenolysis
↓ glucose uptake
Cortisol:↑ gluconeogenesis
↓ glucose uptake
GH: ↓ glucose uptake

• Muscle
o There is an increase ability to reabsorb glucose
• Hormones
o Insulin • Major Actions:
• It is synthesized in the pancreas (mainly by the • For mobilization of Glucose
beta cell of the Islet of Langerhans) o Hepatic Glycogenolysis
• It consist of an alpha and a beta chain linked o Hepatic Gluconeogenesis
by disulfide bonds o Fatty Acid Oxidation
• It is involved in the reuptake and storage of o Ketogenesis
glucose • Inhibits:
• Major actions: o Glucogen Synthesis
• For fuel storage: o Muscle Glucose Uptake
o Glycogen synthesis o Cellular Uptake of Amino Acid, Potassium,
o Muscle glucose uptake Magnesium
o Cellular uptake of amino acid, o Protein Synthesis
potassium, magnesium
o Protein synthesis o Somatostatin
• Inhibits: • Inhibits both the insulin and glucagon
o Lipolysis • Major Actions:
o Ketogenesis • Inhibition of insulin and glucagon secretion
o Hepatic glycogenolysis • Decrease the motility of GIT
o Hepatic Gluconeogenesis • Deacrease GIT secretion, degestion and
o Glucose Release absorption
o Muscle proteolysis • Coordinated the nutrient input in the system
o Glucagon like peptides
o Glucagon • Stimulates insulin release by changing the beta cell
• Synthesized by the alpha cells in the Islets of response to increase in glucose
Langerhans • Decrease of glucagon
• It is stimulated by a decrease in the glucose • Slows gastric emptying
level of the blood • Lowers plasma glucose concentration
o Pancreastatin
• Inhibits insulin secretion
• Involved in autofeedback regulation
46. Define plasma glucose threshold and renal transport maximum for glucose.

• Plasma glucose threshold: The plasma concentration of glucose in which glucose will first appear in the urine. When the plasma glucose
concentration rises above 200 mg/dl, the filtered load of glucose increases to more than 250 mg/min, and a small amount of glucose appears in
the urine.
• Glucose: 100% filterable; reabsorbed in first half of proximal tubule via Na/glucose symport powered by Na gradient generated by Na,K ATPase
(Na-linked secondary active transport)
• Renal transport maximum: maximum rate at which glucose can be transported from the tubules. At around 375 mg/min, all nephrons have
reached their maximum capacity to reabsorb glucose. After this point, glucose cannot traverse across the membrane, and will begin to appear
in urine.
• Definition: maximum rate of reabsorption (or secretion) of a substance
• tubular maximum rate (Tm) for glucose approx. 320 mg/min (may vary between diff. nephrons)
• Filtered load (mg/min) = plasma conc.(mg/ml) X GFR (ml/min)
• If filtered glucose load > Tm for glucose, then excess spills over into urine.
• Plasma glucose threshold = blood glucose conc. at which filtered glucose load starts to exceed Tm for glucose, resulting in glucosuria; Normal
value: 9 - 10 mmol/L (162 - 200 mg/dl).

47. In a normal individual, does glucosuria occurs during an oral glucose tolerance test (OGTT)?

When does glucosuria happen?


When plasma glucose level exceeds renal threshold, Filtrate concentration exceeds Tmax.

The glucose levels of a normal, fasting person ranges from 50-110 mg/dl. The cells are deprived of glucose. Ingestion of glucose will stimulate
insulin to promote uptake of glucose into the cells. Almost all the glucose filtered by the glomerulus will be reabsorbed in the proximal
convoluted tubule by secondary active transport (cotransported with energy from Na+K+ ATPase). The renal threshold is 160 to 180 mg/dl. In
cases of hyperglycemia, reabsorption of glucose stops, thus glucose ix excreted. Glucosuria is seen in patients with diabetes mellitus, in which
there is a lack of insulin or there is insulin resistance. The fasting blood glucose levels for diabetic patients are almost always above 110 mg/dl.
On glucose ingestion, the blood glucose level is much greater than that in normal fasting patients, and glucose level falls back to control level
after 4-6 hours. Saturation of receptors in the kidneys occurs since there are above normal levels of glucose in the kidneys. This occurs at levels
above 11 mmol/L.
When a normal, fasting person ingests 1 gram of glucose per kilogram of body weight, the blood glucose level rises from 90mg/100 ml to 120-
140 mg/100ml, then falls back to below normal within 2 hours. He will not experience glucosuria as the amount of glucose ingested will not
cause the blood glucose level to exceed threshold.
• OGTT
• primarily measures the body's ability to use or metabolize glucose, involves ingestion of set amt. (75-100g) of glucose and measurement of
blood glucose level at set intervals, then compared with the pre-ingestion fasting blood glucose level.
• A normal individual will not experience glucosuria as the amount of glucose ingested is not enough to cause the blood glucose level to
exceed the threshold.

48. Summarize the results of the intervention. Give the physiologic basis for the results.

Parameter Effect Explanation


Urine output ↑ Infusion  ↑ ECF/ICF volume  ↓ sympathetic  ↑ ANP, ↓ ADH & RAA  ↑ Na, H20 excretion

• Infusion  ↑ ECF/ICF volume  ↓ sympathetic  ↑ ANP, ↓ ADH & RAA  ↑ Na, H20 excretion  ↑ urine output
• Initially the 2% glucose is isotonic to ECF; when ingested, the glucose is quickly metabolized and the solution becomes hypotonic. ECF volume
increases, ECF osmolality decreases. Water moves into the cell, also increasing ICF volume. Osmolality decreases. Volume expansion leads to
decreased sympathetic activity, stimulating atrial natriuretic peptide while inhibiting vasopressin and the renin-angiotensin-aldosterone. Sodium
and water excretion is promoted due to decreased RAA activity, resulting in diluted urine.

Lactic Acid (49-51 by Ryan Go 52 by Pasky Atanga)

49. Identify the normal range of pH values, and the upper and lower limits compatible with life. Describe the role of buffers in maintaining pH,
including the roles of the lungs and kidneys.

• Recall: Acidosis (acidaemia) is defined as a disorder with pH in the arterial blood (pHa) less than 7.35, and alkalosis (alkalaemia or baseosis) is
defined as a condition with a pHa larger than 7.45.

• People can suffer from 4 different acid-base disorders: Respiratory alkalosis and acidosis, and metabolic alkalosis and acidosis. Each
is compensated for by the body by complementary actions. That is, if the alkalosis or acidosis is respiratory in nature, then the body’s
response is likewise respiratory. And vice versa.
• Thus, the kidneys serve to compensate for metabolic acid-base imbalances
• While the lungs are responsible for respiratory acid-base imbalances

• pH 6.8 – 7.8 or [H+] 16 – 160 nmol/L


• Normal plasma pH 7.4 (range 7.35 - 7.45)
• Respiratory regulation = ↑ [H+] → ↓ pH  Peripheral chemoreceptors  Respiratory center in medulla  ↑ alveolar ventilation  ↓ CO2 → ↓
[H+] / ↑ pH
• Renal regulation  [H+] secretion = HCO3- reabsorption  HCO3- production
• Proximal Tubule - H+ secretion coupled with Na+ entry, 90% HCO3 resorbed
• Distal Tubule - H+ ATPase, maximal urine acidification pH 4.5
• Titrable acid – Quantify renal acid-base excretion, NaOH added to acidic urine, equal to net H+ excreted
• Urinary Buffers
• Phosphate – Excess H+ combine with PO4, NaPO4 excreted, pK 6.8, dependent on phosphate intake
• Ammonia – Most important quantitatively, Proximal tubule (Glutamine metabolism), Collecting tubules (H+ + NH3  NH4)
• Filtered vs newly formed HCO3- = Secreted [H+], Buffer other than HCO3-

50. Distinguish between the reclamation of filtered bicarbonate and the formation of new bicarbonate.

Since both are bicarbonate (basic) saving, then we can assume that both actions are a means of increasing pH, and therefore occur in order to
balance a patient experiencing:

Metabolic Acidosis

The obvious distinction here is that the former attempts to recycle bicarbonate which has been filtered, while the latter is the generation of new
bicarbonate in the distal tubule; both however work to replenish body buffer stores and function as compensatory (metabolic as opposed to
respiratory) alkalosis.

Reabsorption of bicarbonate in the proximal and distal parts of the each bicarbonate produced in the cell from the CO2 of the tubular fluid,
nephron. one bicarbonate ion diffuses to the interstitial phase and the renal venous
Also within the cell CA facilitates the production of (H+ + HCO3-). For blood back to the body (Fig. 17-4). The cells of the thick ascending limb of
the Henle loop also reabsorb bicarbonate by the same mechanism as
in the proximal tubule
The small residue of bicarbonate enters the distal tubules, where it is
reabsorbed almost totally through a special mechanism independent
of Na+ . In the intercalated cells of the collecting ducts, the
reabsorption is dependent on a proton-K+-ATPase (Fig. 17-4). The
bicarbonate ion crosses the basolateral cell membrane in exchange of
chloride through a chloride-bicarbonate antiporter. - This special
mechanism is most likely ineffective in distal renal tubular acidosis.
Acidosis, which involves the intracellular space and stimulates
production of proton-K+-ATP-ases, also favours H+ -secretion. Hereby,
bicarbonate reabsorption is stimulated, whereas alkalosis inhibits
bicarbonate reabsorption by the opposite mechanisms.
Aldosterone stimulates the proton-K+-ATPases of the intercalated
cells and the Na+-reabsorption/ K+ -secretion of the principal cells.
Both effects favour H+ -secretion and thus bicarbonate reabsorption.

51. Describe net acid excretion by the kidneys, titratable acid, the importance of urinary buffers, and the production and excretion of ammonium.
What do you think will happen to the titratable acid, the importance of urinary buffers, and the production and excretion of ammonium during lactic
acid infusion.

Glutamate metabolism and renal ammonia production. The renal handling of ammonia and the excretion by diffusion trapping is shown.In
the proximal tubules of the kidneys, renal glutamate produces NH4+ and a-ketoglutarate. One molecule of NH4+ is produced by deamination of one
glutamine molecule by the enzyme, glutaminase, and a second by oxidative deamination of glutamic acid forming a -ketoglutarate that is
metabolised. The NH4+ in the proximal tubule cells is in equilibrium with minimal amounts of NH3 at the relatively low pH. The NH4+-secretion into the
tubular fluid makes use of the Na+-H+-antiporter, where NH4+ substitutes H+. The NH4+ passes with the tubular fluid to the thick ascending limb of the
Henle loop, where a major portion is reabsorbed and accumulated in the interstitial fluid (Fig. 17-5).

Secretion of NH4+ in the collecting ducts involves a special mechanism. The NH3 is lipid soluble and easily passes any membrane, so it reaches the
tubular fluid of the collecting ducts and form NH4+ at the low pH (Fig. 17-5). The charged molecule cannot pass the membrane and it is trapped in
the tubular fluid and eliminated in the urine. This diffusion trapping of charged molecules such as NH4+ is called non ionic diffusion - a general
elimination principle for many charged metabolites and drugs. Excretion of NH4+ reduces the excretion of other positive ions.
The a -ketoglutarate is metabolised into bicarbonate. Bicarbonate of the extracellular fluid reacts with H+ from hepatic phosphoric and sulphuric acid
to form carbon dioxide and water. The H2PO4- (and a minimal amount of SO42-) is excreted in the urine. On a mixed diet the production and excretion
of non-volatile acids and bases results in a net excretion of acids equal to the daily net production of non-volatile acids.
With a urine pH of 6.5, organic acids such as lactic acid, b-hydroxybutyric acid, pyruvic acid etc., are present in the base form (RCOO- of Fig. 17-6).
Most of the phosphoric acid is H2PO4-, and almost all ammonia is in the NH4+ form.
Excretion flux for organic bases (RCOO-), titratable acid (H2PO4-), and NH4+ in normal daily urine.
This is not so in an alkaline urine. At a urinary pH of 8, there is 5% NH3 of the total, just as in ileum and colon, where the pH is also 8.
The high pK (=9.3) of NH3/NH4+ has the consequence that in gastric juice with a pH of 1, the (pH-pK)- difference is -8.3, so virtually all ammonia must
be NH4+. Even in body fluids with a pH of 7.3 the NH3/NH4+ ratio is 1/100.

Lactic acidosis a result of lactic acid infusion can be caused by increased lactic acid production during exercise, shock, anoxia or following cardiac
arrest. Another type of lactic acidosis is caused by decreased hepatic lactate metabolism - often drug-induced.

The body therefore compensates for this acidosis through the aforementioned compensatory behaviors.
52. Summarize the results of the intervention. Give the physiologic basis for the results
Parameter Effect Explanation
Pupil size ↑ Sympathetic stimulation of the alpha receptors of readial muscle lead to mydriasis (increase pupil size)
Cardiac rate ↑ ↑H+(↓pH) stimulation of ventilatory center, ↑RR CO2, ↓ pH
MAP ↑ Same as cardiac rate
Respiratory rate ↑ Same as cardiac rate
Intrapleural pressure ↑ Stimulation of resp. center diaphragm leads increased effort of breathing & increased intrapleural
pressure.
Intestinal movement ↓ Duet to sympathetic stilmuatlon and shunting of blood away from splancnic circulation, there’s
decreased peristalsis and muscle tone.
Urine output ↓ Increased sympathetic activity constricts vessels, increased MAP decreased renal blood flow and
decreased urine output.

• ↓ pH = peripheral chemoreceptors  ↑ TV & RR  ↑ Intrapleural pressure;


• Sympathetic effects = ↑ pupil size, HR; ↓ intestinal movement
• ↓ pH = ↑ H secretion = ↑ Na reabsorption = fluid expansion = ↑ ECV = ↑ MAP, ↓ urine output

Partial Airway Occlusion (53 by Bianca Arambulo 54-55 by Antoinette dela Pena 57-58 by Nicole Cacino 56, 59-60 by Pasky Atanga)

53. Diagram & discuss the lung volume, tracheal pressure, alveolar pressure, and pleural pressure during a normal quiet breathing cycle. Identify
the different phases of the respiratory cycle. Relate the pleural and airway pressure values to the movement of air.

*Air moves from an area of greater pressure to lesser pressure

Inspiration: The diaphragm & inspiratory muscles contract, while the


thorax expands, creating a more negative intrapleural pressure. Since
the lungs volume increases, tranpulmonary pressure increases. While
the alveolar pressure becomes more negative than the atmospheric
pressure. Air goes to the alveoli.

Expiration: Diaphragm & inspiratory muscles stop contracting, chest


wall recoils back. Intrapleural pressure increases (less negative),
transpulmonray pressure increases. Alveolar pressure becomes
positive, and gas flows from greater (alveolus) to lesser
( atmospheric) pressure.
Patm= atmospheric pressure
* The lung requires a positive transpulmonary pressure in order to Palv= alveolar pressure
increase its volume. The lung assumes its smallest size when the Pip= intrapleural pressure
transpulmonary pressure is zero. Transpulmonary pressure= Palv-Pip

54. State the relationship of the lung volume with the compliance/ elastance of the following a) lungs, b) chest wall and c) respiratory system.

Compliance - the extent to which the lungs expand for every unit
increase in transpulmonary pressure
Elastance - is the ability of the lungs or chest wall to return to its
resting state after distension; counteracts the force that tends to
distend or expand the lungs
Two components:
a. lung tissue itself
- collagen and elastin
b. surface tension
- fluid lining the alveolar walls
- ~2/3 of the total elastic forces
lung volume low ↑ high
compliance high ↓ Low
elastance low ↑ high

55. Discuss the factors that determine airway resistance. Discuss the factors that govern the luminal radius of an airway.

Airway resistance is the opposition to flow caused by the forces of friction. It is defined as the ratio of driving pressure to the rate of air flow.
Resistance to flow in the airways depends on whether the flow is laminar or turbulent, on the dimensions of the airway, and on the viscosity of the
gas.

For laminar flow, resistance is quite low. That is, a relatively small driving pressure is needed to produce a certain flow rate. Resistance during
laminar flow may be calculated via a rearrangement of Poiseuille's Law :

The most important variable here is the radius, which, by virtue of its elevation to the fourth power, has a tremendous impact on the resistance.
Thus, if the diameter of a tube is doubled, resistance will drop by a factor of sixteen.
For turbulent flow, resistance is relatively large. That is, compared with laminar flow, a much larger driving pressure would be required to produce
the same flow rate. Because the pressure-flow relationship ceases to be linear during turbulent flow, no neat equation exists to compute its
resistance.
While a single small airway provides more resistance than a single large airway, resistance to air flow depends on the number of parallel pathways
present. For this reason, the large and particularly the medium-sized airways actually provide greater resistance to flow than do the more numerous
small airways.
Airway resistance decreases as lung volume increases because the airways distend as the lungs inflate, and wider airways have lower resistance.
Tone of smooth muscle that surrounds airways affects their diameter and therefore resistance. Smooth muscle reacts to both parasympathetic and
sympathetic nervous activity. Stimulation of parasympathetic fibers releases Ach which causes smooth muscle contraction (thus constricting airways)
while sympathetic activation releases NE which relaxes smooth muscle causing dilation of the airway. Epinephrine and beta 2 agonists are powerful
bronchodilators. In addition, there are a large number of enteroendocrine cells in respiratory system whose secretions alter airway diameter.
Substance P is potent bronchoconstrictor and stimulator of mucus secretion. VIP has the opposite effect

56. Compare the excitation contraction coupling of the skeletal and bronchial smooth muscles. Discuss the consequences of the neostigmine
administration on the bronchial smooth muscle tone. Explain the changes seen in the intrapleural pressure and respiratory rate during neostigmine
administration.

Skeletal Muscle Bronchial Smooth Muscle


L-type Calcium channel (DHP receptor) Calcium entry via voltage gated Calcium channels
in T- tubule membrane coupling to Calcium and IP3 mediated calcium release from SR
Excitation Contraction Coupling
Calcium release channel ( Ryanodine Calcium entry through store operated calcium
receptor) in SR channels
Calcium sensor Troponin Calmodulin
Breakdown of Ach by
Termination of Contraction Myosin light chain phosphatase
acetylcholinesterase

What is Neostigmine? is a parasympathomimetic


Reversible cholinesterase inhibitor
-inhibits cholinesterase which destroys the acetylcholine at nerve endings
-indirectly stimulates both cholinergic and muscarinic receptors
-mimics the action of the parasympathetic nervous system

Discuss the consequences of the neostigmine administration on the bronchial smooth muscle tone

Parameters Effect
AcetylCholinesterase activity Decreased
Acetylcholine Increased
Bronchial Smooth Muscle Contraction Increased
Surface Area Decreased
Respiratory Rate Increased
Intrapulmonary Pressure Decreased

Explain the changes seen in the intrapleural pressure and espiratory rate during neostigmine administration.

Intrapulmonary Pressure:
Neostigmine administration Parasympathetic stimulation Formation of IP3 and DAG, increased intracellular calcium bronchoconstriction
Decrease airway caliber INCREASED AIRWAY RESISTANCE DECREASED INTRAPLEURAL PRESSURE

Respiratory Rate:
Neostigmine administration Parasympathetic stimulation bronchoconstriction
INCREASED AIRWAY RESISTANCE LOW VENTILATION, LOW VQ (hypoxemia, hypercapnia) Peripheral and central chemoreceptor RESPIRATORY
RATE INCREASES

57. Give the effect of the changes in airway resistance and respiratory system compliance with the ventilation of the respiratory units.

VENTILATION = COMPLIANCE/AIRWAY RESISTANCE

• Compliance, ventilation

• Airway resistance, ventilation

58. Discuss the pressure volume loop during tidal breathing and relate the components of work of breathing. Speculate on what will be the
pressure volume of the dog once the airway was partially occluded.

*Please refer to CHAPTER 26 of Berne and Levy, pp. 477-478


WORK OF BREATHING = Pressure x ∆Volume
Pressure
• Change in transpulmonary pressure needed to overcome the work of breathing
∆ Volume
• Volume of air moving into and out of the lungs
• Also known as TIDAL VOLUME
What “work” is done while breathing?
1. Use of respiratory muscle expends energy
2. In order to move both the lungs and the chest wall, certain mechanical properties of the lung must be overcome
2.1. ELASTIC WORK
2.1.1. Elastic recoil of the lungs and chest wall
2.1.2. Surface tension in the alveoli
2.2. RESISTIVE (NONELASTIC) WORK
2.2.1. Tissue resistance
2.2.2. Airway resistance
Pressure-volume loop
RESPIRATORY CYCLE OF A NORMAL LUNG
• Estimation of mechanical work by measuring pressure and volume changes
during a respiratory cycle
• ACB: static inflation-deflation curve
• OAECD: total mechanical workload
• OABCD: work necessary to overcome elastic resistance
• AECF: work necessary to overcome nonelastic resistance
• AECB: work necessary to overcome nonelastic resistance during inspiration
• ABCF: work necessary to overcome nonelastic resistance during expiration
Pressure-volume loop of a partially occluded airway

• LUNG WITH INCREASED AIRWAY RESISTANCE


(OBSTRUCTIVE)
• Increased airway resistance, increased work of breathing
• E.g. asthma, chronic bronchitis, emphysema
• FATTER, WIDER CURVE
• OAECD: increase in total inspiratory work
• DFO: Greater negative pleural pressures needed to
maintain proper inspiratory flow rates (wider space
occupied by inspiratory curve)
• Increase in positive pleural pressure during exhalation
(wider space occupied by expiratory curve; reaches
positive side of x-axis)
• Increased resistance
• Increased expiratory workload
• Stored elastic energy (see area ABCF in curve for normal
lung) insufficient; additional energy needed for
exhalation
• Recruitment of respiratory muscles
• Exhalation no longer a passive process
• Respiratory muscles can perform increased work over long periods of time
• Will eventually experience fatigue; result in respiratory failure
• Work of breathing influenced by breathing patterns (compensatory mechanism)
• Increased WOB when respiratory rate is increased (as observed in the dog when airway was partially occluded)
• Increase in minute ventilation requires more flow resistance forces to overcome
• Adopt certain respiratory patterns to minimize WOB
• Those with obstructive lung disease breathe more slowly and deeply

59. Describe pattern of airflow from the trachea to the alveoli. Relate the pattern of airflow to the Reynolds’s number.

Reynold’s number = Re = 2rvd


ŋ
Three types of Flow parterns: Turbulent, Transitional and Laminar flows.

Area Airway Caliber Airflow pattern


Trachea Turbulent
Bronchi Intermediate (most of tracheobronchial tree)
Bronchioles Laminar
Alveoli No airflow pattern here.

Trachea- Main bronchi-Interlobar bronchi- Segmental bronchi- Non- FLOW


respiratory bronchioles- Respiratory bronchioles- Alveoli  Occurs at high flow rates
*Air flows to airways of decreasing sizes  Complete disorganization of streamlines
Re = 2 radius•average velocity•gas density  Molecules move laterally, collide with one another, & change
viscosity velocities
Laminar Flow at <2000  Trachea
Transitional flow in between 2000-3000 TRANSITIONAL
Turbulent flow at >3000 FLOW
 characteristics of both laminar and turbulent flow
 edges of the fluid flow (laminar)
 center of the flow remains turbulent
 dependent on viscosity & density of gas
 Large part of tracheobronchial tree
LAMINAR FLOW
 Organized
 Streamlines are parallel
 Center: faster; sides: slower
 Capable of sliding over one another
 Peripheral airways
TURBULENT
60. Summarize the results of the intervention. Give the physiologic basis for the results.
Intervention:
Partial occlusion of tracheal lumen (50%) due to material within the Sympathetic Structure Parasympathetic
lumen. Rate increased Heart Rate decreased
Bronchial muscle Lungs Bronchial muscle
Partial occlusion Intervention summary: relaxed contracted
Stress (sympathetic system activation) – stimulated by hypoxia
Hypoventilation (hypoxia and hypercapnia)
Hypoxemia  Peripheral chemoreceptors 
Pupil dilation
Motility reduced
Eye
Intestine
Pupil constriction
5
Digestion increased
 Sphincter closed Bladder Sphincter relaxed
CPG  increase ventilation Decreased urine
Hypercapnia  central chemoreceptors via Kidneys Increased urine secretion
 secretion
change in CSF pH & peripheral
chemoreceptors CPG  increase
ventilation

Parameter Effect Explanation


Pupil size ↑ Sympathetic stimulation of the alpha 1 receptors of readial muscle lead to mydriasis (increase pupil size)
Cardiac rate ↑ Decreased PO2 causes increased depth of breathing, causes less venous return to heart and less
MAP ↑ filling pressure. This will inhibit baroreceptors and wil result in increased heart rate, increased
cardiac output and MAP
Respiratory rate ↑ Increased PCO2 stimulates central chemoreceptors in ventrolateral medulla. Decreased PO2
Intrapleural pressure ↓ stimulates peripheral chemoreceptors in aortic and carotid bodies. Response is hyperventilation
and increase in respiratory depth (increased tidal volume).
Intestinal movement ↓ Sympathetic stimulation of Alpha 2 and Beta 2 receptors inhibits peristalsis and increases tone of
sphincter
Urine output ↓ Sympathetic activity activates RAAS system, causes increase in sodium and water reabsorption in
the kidneys
• ↓ pH= peripheral chemoreceptors  ↑ TV & RR  ↑ Intrapleural pressure;
• Sympathetic effects = ↑ pupil size, HR; ↓ intestinal movement
• ↓ pH = ↑ H secretion =↑ Na reabsorption = fluid expansion = ↑ ECV = ↑ MAP, ↓ urine output

Mechanisms of compensation
1.Immediate (even during first breath in which resistance is applied)
- muscle spindle  inspiratory muscles fail to shorten
- DRG
2. Develops about 90 seconds & overacts for a similar period when the resistance is removed
- elevation of arterial PCO2
- Chemoreceptors
 Hypoxemia  Peripheral chemoreceptors  CPG  increase ventilation
 Hypercapnia  central chemoreceptors via change in CSF pH & peripheral chemoreceptors CPG  increase ventilation

Carbogen (61 by Mary Grace Causapin 62 by Irish Chang 63-64 by Maylyn Cauilan 65-66 by Mary Grace Causapin)

61. Define partial pressure and fractional concentration as they apply to gases in air, airways, alveoli, pulmonary capillary blood, arterial blood and
venous blood. List normal values for each for O2, CO2, and N2. What happens to these values at high altitude and deep sea?
Partial pressure - The pressure that one component of a mixture of gases would exert if it were alone in a container
Gas Atmospheric gas composition partial Total pressure at sea level
pressure (%) ATM pressure = 760 mm Hg
Nitrogen 78.62 597.5
Oxygen 20.84 158.4
Carbon Dioxide 0.04 0.3

62. Define percent hemoglobin saturation, oxygen tension, oxygen content as they pertain to blood.
• Percent Hemoglobin saturation
o The chemical binding of O2 to hemoglobin occurs in the lung and this HgbO2 complex is the major transport mechanism for
oxygen
o number of O2 molecules bound to hemoglobin is dependent on the partial pressure of O2 in the blood. the partial pressure of O2
rises, hemoglobin saturation increases.
o OXYGEN HEMOGLOBIN DISSOCIATION CURVE: At partial pressures below 60 mm Hg, oxygen readily binds to
hemoglobin as the PO2 increases. At a PO2 of 60 mm Hg, hemoglobin is 90% saturated; increases in PO2 above this level will
influence hemoglobin saturation only slightly. Specifically, increasing the PO2 from 60 to 100 mm Hg will increase the
hemoglobin saturation by only 7%. The clinical significance of the flat portion of the oxy hemoglobin dissociation curve is that a
drop in PO2 from about 100 mm Hg to about 60 mm Hg still results in a hemoglobin saturation of more than 90%, which virtually
ensures adequate O2 transport. Also, increasing the PO2 above 100 mm Hg has little effect on the oxygen content in the blood,
because hemoglobin is already fully saturated. In the steep portion of the curve, blood oxygen content and thus oxygen delivery
to the tissue are significantly compromised when the PO2 falls below 60 mm Hg. The clinical significance of this portion of the
curve is that a large amount of O2 is released from hemoglobin with only a small change in PO2. This response facilitates the
diffusion of O2 to the tissue. The point on the curve at which 50% of the hemoglobin is saturated with O2 (two oxygen molecules
on one Hgb molecule) is called the P50
o RISE in 2-3HCOT will make Hgb REJECT O2 leading to REDUCED Hgb saturation. This will casue a RIGHT shift of the Hbg-
O2 dissociation curve and will manifes as a RISE in P50; LOW levels of 2-3 HCOT will make Hgb LOVE O2 thus resulting in
higher Hgb-O2 saturation, causing a shift to the LEFT of the Hgb-O2 dissociation curve manifested by a LOWER P50.
• Oxygen tension (PO2)
o The composition of a gas mixture can be described in terms of either the gas fractions or the corresponding partial pressure.
o When Boyle's gas law is applied, the sum of the partial pressures (in mm Hg) or of the tensions (in torr) of a gas must be equal to
the total pressure. Thus, at sea level, where the partial pressure is 760 mm Hg, the partial pressures of the gases in the
atmospheric air [also known as the barometric partial pressure (Pb)] are

The second important principle is that the partial pressure of a gas (Pgas) is equal to the fraction of gas in the gas mixture (Fgas)
times the total or ambient (barometric) pressure.

Ambient air is composed of approximately 21% oxygen and 79% nitrogen. Therefore, the partial pressure of oxygen in ambient
air (PO2) is

o When there is both normal alveolar ventilation and normal alveolar capillary blood flow (normal alveolar perfusion), exchange of
oxygen and carbon dioxide through the respiratory membrane is nearly optimal, and alveolar Po2 is normally at a level of 104
mm Hg, which lies between that of the inspired air (149 mm Hg) and that of venous blood (40 mm Hg). Likewise, alveolar Pco2
lies between two extremes; it is normally 40 mm Hg, in contrast to 45 mm Hg in venous blood and 0 mm Hg in inspired air.
Thus, under normal conditions, the alveolar air Po2 averages 104 mm Hg and the Pco2 averages 40 mm Hg.

o
• Oxygen content (in mL)
o Amount of oxygen carried by blood in mL
o Not synonymous to O2 tension
o (O2 in physical solution) + (bound to Hgb)
Physical solution = 0.003ml/dl x PaO2
Bound to hemoglobin = (Hgb grams /dl) x (1.34 ml/gram) x (O2 sat)
o Determinants
 Directly proportional to minute ventilation, alveolar ventilation and hemoglobin level
 Inversely proporitional to dead space ventilation
 Position of O2 dissociation curve
63. List the normal alveolar, arterial, and mixed venous blood gas values for Po2, Sato2, Pco2, HCO3, and pH.
Alveolar Arterial Venous
PO2 104 mm Hg 95 mm Hg 40 mm Hg
SatO2 98% 97.5% 75%
PCO2 40 mm Hg 40 mm Hg 45 mm Hg
HCO3 24 mEq/L 24 mEq/L 21.9 mEq/L
pH 7.40 7.40 7.31
64. Discuss the hemoglobin oxygen dissociation curve. Enumerate the conditions that may alter its normal position. Name the five causes of
hypoxemia. Identify the factors that determine O2 delivery.

Causes of Hypoxemia
1. Shunt – inc. P(A-a)O2 PaCO2 normal
- cannot be corrected with supplemental oxygen

a. Anatomic – portion of blood bypasses lungs (e.g. bronchial


vein-pulmonary vein, thebesian vein)
b. Physiologic – portion of blood goes through normal
vasculature but does not come into contact with alveolar air
2. Low Inspired Oxygen (PIO2) – normal P(A-a)O2, dec. PaCO2 due to
hyperventilation
3. Low V/Q – inc. P(A-a)O2, normal PaCO2
- most common cause of hypoxemia
4. Hypoventilation – inc. PaCO2, normal P(A-a)O2
- mechanical ventilation required to eliminate hypercapnia
5. Diffusion abnormality/impairment – inc. P(A-a)O2 with exercise, normal
PaCO2
- rare

Hgb-O2 Dissociation curve = relates O2 partial pressure and %saturation of Hgb


• Reject O2 = Rightward shift = Rise in P50 & HCOT 2,3DPG (H+, CO2, Temp, 2,3DPG) = Reduced Hgb-O2 saturation
• Loves O2 = Leftward shift = Lower P50 & HCOT 2,3DPG = CO (carbon monoxide) poisoning, fetal Hgb (vs maternal)
• Anemia = usually does not affect curve
• Carbon Monoxide = shifts the O2-Hgb dissociation curve to the left

Determinants of O2 delivery

O2 delivery to tissue = Cardiac Output x O2 content


O2 Del. = C. O. x (PaO2 x 0.003 mL/dL) + (Hg x 1.34 mL/dL x O2 saturation)

65. Discuss the peripheral and central chemoreceptor pulmonary reflexes.

Central Ventrolateral medulla Sensitive to H+ in CSF by CO2 changes


Peripheral Carotid bifurcation or aorta Sensitive to ↓ PaO2 and slightly to PCO2 and H+

Chemoreceptors
•central and peripheral chemoreceptors. ↓ Inspired PO2

•They respond to changes in the PCO2, pH, and PO2 of arterial blood.
↓ Alveolar PO2
Central Chemoreceptors in the medulla monitor the pH associated with CO2
levels within the cerebrospinal fluid in the fourth ventricle
↓ Arterial PO2
Chemoreceptors
•The Peripheral Chemoreceptors are found in two locations:
1. the aortic bodies within the aortic arch
Peripheral Chemoreceptors
2. the carotid bodies at the bifurcation of the common carotid arteries
↑ Firing
•They are stimulated by Reflex via the medullary
–Decreased PO2 (hypoxia) respiratory neurons
–Increased hydrogen ion concentration (metabolic acidosis)
–Increased PCO2 (respiratory acidosis) Respiratory Muscles
↑ Contractions

Central Chemoreceptors: Effect of PCO2


• The most important factor controlling the rate and depth of breathing is the
effect of carbon dioxide on the central chemoreceptors.
• Carbon dioxide readily diffuses from the blood into the cerebrospinal fluid ↑ Ventilation
in the fourth ventricle. Here, carbon dioxide combines with water to form
carbonic acid, which dissociates into hydrogen ions and bicarbonate ions.
Most of the hydrogen ions within the cerebrospinal fluid are derived from this
chemical reaction:
CO2 + H2O H2CO3 H+ + HCO3- Return of alveolar and
• The hydrogen ions stimulate the central chemoreceptors, which send nerve arterial PO2 to normal
impulses to the respiratory centers in the medulla.
• As carbon dioxide increases, so does the number of hydrogen ions, which in
turn lowers the pH. The central chemoreceptors actually respond to this pH change caused by the blood PCO2.

Predict the Effect of Increased PCO2


• What will happen to the breathing rate and depth if the arterial PCO2 increases?
• An increase in the PCO2 in the blood leads to an increase in hydrogen ions in the cerebrospinal fluid, decreasing the pH.
• The central chemoreceptors fire more frequently, sending more nerve impulses to the respiratory centers, which in turn send more nerve impulses
to the respiratory muscles.
• This results in an increased breathing rate and depth, allowing more carbon dioxide to be exhaled, returning the blood PCO2 to normal levels.
the amount of oxygen loaded into the blood.
Peripheral Chemoreceptors: Effect of pH Changes
• The peripheral chemoreceptors also respond to pH changes caused by PCO2
changes, however they directly monitor changes in the arterial blood, not the
cerebrospinal fluid as the central chemoreceptors do.
• The role of the peripheral chemoreceptors:
• Increased carbon dioxide levels in the arterial blood result in decreased
blood pH, which stimulates the peripheral chemoreceptors.
• They respond by sending more nerve impulses to the respiratory centers,
which stimulate the respiratory muscles, causing faster and deeper
breathing.
• More carbon dioxide is exhaled, which drives the chemical reaction to the
left and returns the PCO2 and pH to normal levels.

Breathing gas mixture containing


CO2

↑ Alveolar PCO2

↑ Arterial PCO2

↑Brain ECF PCO2


↑ Arterial [H+]
↑ Brain ECF [H+]

Peripheral chemoreceptors
Peripheral Chemoreceptors: Effect of PO2
↑Firing Central chemoreceptors
• The peripheral chemoreceptors also respond to acids such as lactic
↑ Firing
acid, which is produced during strenuous exercise:
• Active muscles produce lactic acid, which enters the blood,
releases hydrogen ions, and lowers the pH. Respiratory Muscle
• The decreased pH stimulates the peripheral chemoreceptors to ↑ Contraction
send more nerve impulses to the respiratory centers, which stimulate
the respiratory muscles to increase the breathing rate and depth.
• More carbon dioxide is exhaled, lowering the PCO2 in blood, driving ↑ Ventilation
the chemical reaction to the left, and lowering hydrogen ion levels.
• The peripheral chemoreceptors also monitor arterial PO2, however,
the arterial PO2 must drop below 60 millimeters of mercury before Return of alveolar and arterial PCO2 to normal
the chemoreceptors respond.
• The normal alveolar PO2 of about 100 millimeters of mercury
results in 98% hemoglobin saturation in the blood.
• If the PO2 drops to 60 millimeters of mercury, hemoglobin is still
Return of brain ECF PCO2 to normal
90% saturated. Return of alveolar and
• Any increased ventilation in this range of PO2's results in only a arterial [H+] to normal
small increase in the amount of oxygen loaded into the blood.
• However, at very high altitudes, the alveolar PO2 may fall to 40 Return of brain ECF [H+] to normal
millimeters of mercury and hemoglobin will be only 75% saturated. At
this point, increased ventilation will make a dramatic difference in

66. Summarize the results of the intervention. Give the physiologic basis for the results.

Carbogen
has higher than normal levels of carbon dioxide. When taken intentionally to trigger an abreaction it can also cause feelings of panic and anxiety.

Carbogen is inhaled, the increase of carbon dioxide in the lungs causes a perception of physiological hyperventilation.

hyperventilation (or overbreathing) is the state of breathing faster and/or deeper than necessary, thereby reducing the carbon dioxide concentration
of the blood below normal.

Introduction of carbogen will be detected as increase in PCO2 and Decrease in PO2 by the chemoreceptors that will stimulate the vasomotor region
in the medulla. It elicits a constellation of sympathetic response.

Pupil Size
Vasomotor center (medulla)=>Sympathetic Discharge =>Stimulate the
α adrenergic receptors of the iris (radial muscle) -->Dilation of pupil
Cardiac rate:
↑ Increase in CO2 and H+ Stimulate central chemoreceptors --> ↑
sympathetic stimulation
--> ↑ contraction of cardiac muscles --> ↑ heart rate and force of
atrial and ventricular contractions --> ↑ in Cardiac output

Mean Arterial Pressure


COxTPR = MAP
Increase in cardiac output will increase MAP

Respiratory rate
↑ in CO2 and H+ --> Stimulate central chemoreceptors --> Muscles
of inhalation and exhalation to contract more forcefully and more
frequently (hyperventilate) --> ↑ in Respiratory rate

Intrapleural pressure
↑ CO2 and H+ --> ↑ ventilatory drive --> ↑ tidal volume --> ↑
intrapleural pressure

Intestinal movement
↑ in CO2 and H+ Stimulate contraction of of sphincter --> ↓ in
motility and tone --> ↓ intestinal movement
Urine output
↑ in CO2 and H+ --> Stimulate the constriction of the blood vessels
(kidney arterioles) --> ↓ GFR and inhibition to the detrussor muscle
--> ↓urine volume
Carbogen intervention (5% CO2- 95% O2 mixture gas tank) = High CO2 and high O2
Effects: ↑ CO2 = ↑ H+ in blood = ↑ sympathetic / ↓ parasympathetic
Parameter Effect Explanation
Pupil size Sympathetic dominance (α adrenergic)
Cardiac rate Sympathetic dominance
MAP ↑ HR x SV = CO x TPR = MAP, so ↑ HR  ↑ MAP
Respiratory rate Central chemoreceptor stimulation (via CO2 and H+)  ↑ ventilation
Intrapleural pressure Central chemoreceptor stimulation (via CO2 and H+)  ↑ ventilation  ↑ TV
Intestinal movement ↓
Sympathetic dominance
Urine output

Pnuemothorax (67-68 by Irish Chang 69-70 by Marryann Chiombon)

67. Discuss the different lung volumes and capacity. What will be the effect of pnuemothorax on these volumes and capacities.

• Tidal volume (TV) = volume of air entering or leaving the lungs during a single breath.
• Inspiratory reserve volume (IRV) = volume of air which can be inspired over and above the resting tidal volume
• Expiratory reserve volume (ERV) = volume of air which can be expired after a normal expiration.
• Residual volume (RV) = volume of air remaining in lungs after maximal expiration, can be estimated as 25% of the vital capacity
• Inspiratory capacity (IC) = maximum volume which can be inspired after a normal expiration = IRV + TV
• Vital capacity (VC) = maximum volume that can be expired after a maximal inspiration = IRV + TV + ERV
• Functional residual capacity (FRC) = volume of air left in the lungs after a normal expiration = ERV + RV
• Total lung capacity (TLC) = volume of the lungs when fully inflated = VC + RV (or 1.25 x VC)
• Pneumothorax – air in pleural space  lung volumes are not equal, lung collapse = ↓ volumes & capacities BUT RV is maintained; low FVC,
normal or low FEV1, FEV1/FVC > .75

68. Review the different types of VQ relationships and its corresponding O2 & CO2 tensions.

V/Q (Ventilation-Perfusion) = O2 delivery to alveoli & CO2 elimination from alveoli / O2 elimination from alveoli & CO2 delivery to alveoli
Types VQ O2 CO2
Infinity ∞ ↑ ↓ Lung Zone V/Q PAO2 PCO2
High >1 ↑ ↓ Upper High ↑ ↓
Ideal 1 104 40 Middle Almost ideal 104 40
Low <1 ↓ ↑ Lower (V/Q is 3x/8x) Low ↓ ↑
Zero 0 ↓ ↑

VQ= 0
The air in the alveolus comes to equilibrium with the blood oxygen and carbon dioxide because these gases diffuse between the blood and
the alveolar air. Because the blood that perfuses the capillaries is venous blood returning to the lungs from the systemic circulation, it is the
gases in this blood with which the alveolar gases equilibrate. In Chapter 40, we will learn that the normal venous blood ([vbar]) has a Po2
of 40 mm Hg and a Pco2 of 45 mm Hg. Therefore, these are also the normal partial pressures of these two gases in alveoli that have blood
flow but no ventilation.

VQ = infinity
The alveolar air becomes equal to the humidified inspired air. That is, the air that is inspired loses no oxygen to the blood and gains no
carbon dioxide from the blood. And because normal inspired and humidified air has a Po2 of 149 mm Hg and a Pco2 of 0 mm Hg, these
will be the partial pressures of these two gases in the alveoli.

69. Identify the average V/Q ratio in a normal lung. Explain how V/Q is affected by the vertical distribution of ventilation and perfusion in the
lung.

Ave. ventilation in a normal lung: 4L/ min


Ave. perfusion in a normal lung: 5L/min
Ave. V/Q ratio in a normal lung: 0.8

*Lower lung zones are better ventilated (3x) & better perfused (8x)
[Gravity affects transpulmonary & intrapleural pressure making it less (-)]

70. Summarize the results of the intervention. Give the physiologic basis for the
results. Speculate on the VQ relationship on the normal and abnormal lung. What are the consequences on the O2 tension, hemoglobin O2
saturation and O2 delivery to the tissues?

Pneumothorax  Lung collapse  ↓ Lung volume  ↓ ventilation (dec. lung compliance)  ↓ PAO2 (bec. Of dec. ventilation)  ↓ Hgb-O2 sat
(Hgb rejects O2)  ↓ O2 delivery to tissues (Hgb fails to deliver O2)  ↑ PCO2 (bec. of dec. O2) ↑ H+ (bec. of inc. CO2)  peripheral
chemoreceptors (bec. of ↑ CO2 & ↑ H+)  ↑ RR (caused by chemoreceptors)  ↑ intrapleural pressure (caused by ↑ RR) *making it more (-)

sympathetic responses (↑ cardiac rate, ↑ MAP, ↑ renin-angiotensin release leading to ↑ water retention or ↓ urine output

Vagus Central Stump (71-72 by Donnabelle Chu)

71. Enumerate the origin of the different sensory fibers conducted by the vagus nerve.

The Vagus nerve is a mixed cranial nerve that is distributed from the head and neck into the thorax and abdomen. Its sensory axons arise
from the skin of the external ear, a few taste buds, baroreceptors and chemoreceptors in the carotid sinus and carotid body, and visceral
sensory receptors in most organs (thoracic and abdominal). It is composed of parasympathetic fibers and supplies 75% of parasympathetic
action to the body.

The different sensory fibers are:

• General Visceral Afferents (GVA)


• Found within the inferior ganglion
• Conveys information about pressure and volume
• General Somatic Afferents (GSA)
• Found in the superior ganglion
• GVA fibers -> medulla -> form solitary tract -> synapse with neurons of solitary nucleus -> project to different neurons -> nucleus of
the vagus nerve which acts on the different organs

• Nucleus ambiguous- motor nucleus of the vagus nerve (modifies activity of cardiac muscle)

• Dorsal Nucleus of the Vagus – largely secretomotor (activates glands) in the thoracic and abdominal cavity

• Nucleus of the tract of solitarius – sensory fibers from the pharynx, larynx, and esophagus

72. Summarize the results of the intervention. Give the physiologic basis for the results based on your answer to the previous guide question.

Parameter Effect Structure Sympathetic Stimulation Parasympathetic Stimulation


Pupil size Increased Iris (eye muscle) Pupil dilation Pupil constriction
Cardiac rate Increased Salivary Glands Saliva production reduced Saliva production increased
MAP Increased Oral/Nasal Mucus production reduced Mucus production increased
Mucosa
Respiratory rate Increased Heart Heart rate & force increased Heart rate & force decreased
Intrapleural Increased Lungs Bronchial muscle relaxed Bronchial muscle contracted
pressure
Intestinal decreased Stomach Peristalsis reduced Gastric juice secreted; motility increased
movement
Urine output decreased Small Intestines Motility reduced Digestion increased
Large Intestine Motility reduced Secretions & motility increased
Liver Increased conversion of glycogen to glucose
Kidney Decreased urine secretion Increased urine secretion
Adrenal Medulla Norepinephrine & epinephrine secreted
Urinary Bladder Wall relaxed; Sphincter closed Wall contracted; Sphincter relaxed

As discussed in previous labcons, a central stump can create a sympathetic effect to the body thus increasing the results except for
intestinal movement and urine output which it decreases. Physiologic basis of the sympathetic response was already discussed in the
stimulation of the sciatic nerve. After stimulation of the vagus central stump, parasympathetic fibers and effect will predominate.
Parasympathetic nervous system uses acetylcholine as its primary neurotransmitter therefore it has cholinergic receptors. Cholinergic
receptors are divided into nicotinic and muscarinic receptors. Nicotinic responses are of fast onset, short duration and excitatory in
nature. Muscarinic responses are diverse, slower, and longer-lived inhibitory.

 Muscarinic receptors
 M1 receptors: producing slow excitation of ganglia
 M2 receptors: causing decrease in cardiac rate and force of contraction
 M3 receptors: causing secretion and contraction of visceral smooth muscle

When ACh binds to a muscarinic receptor, it causes K+ ions to leave the muscle, hyperpolarizing the membrane &causing an IPSP. It
decreases action potential of the tissue therefore causing it to counteract the effect of the sympathetic nervous system.

 Muscarinic receptors are G- protein coupled receptors which causes activation of phospholipase C (formation of IP3
and DAG as 2nd messenger), inhibition of adenyl cyclase, and activation of K channels or inhibition of Ca channels

It can also be caused by the baroreceptor reflex. When the afferent fibers of the vagus sensed, for example, the increased cardiac rate, it
activates the solitary nucleus which then activates the dorsal nucleus and nucleus ambiguus of the vagus nerve. These nuclei would
activate preganglionic and postganglionic receptors to cause the decrease in heart rate.

Vous aimerez peut-être aussi